NOTEPAD
Results
of 200 questions answered correctly

You have reached of 200 points, ( %)

Your time

Question 1 of 200

1.

A  man  suffering  from  a hereditary disease  married a healthy  woman.  They got  5 children,  three  girls and  two  boys. All  the  girls  inherited their  father’s  disease. What is the type of the disease inheritance?

Explanation

 image

        X-linked dominant: it can be transmitted through both parents. Mothers transmit to 50% of daughters and sons; fathers transmit to all daughters but no son (father-daughters; no generation is left out).

X-linked recessive: sons of heterozygous mothers have a 50% chance of being affected. No male-to-male transmission. Recessive genetic diseases skips generations.

Y-linked: father to all sons; no female involvement (mother or daughter)

Autosomal dominant and recessive

2.

Characteristic   sign   of   glycogenosis is  muscle   pain   during   physical   work. Blood examination reveals usually hypoglycemia. This  pathology  is caused by congenital deficiency  of the  following enzyme:

Explanation

1.     Glycogen phosphorylase is the rate-determining enzyme in Glycogenolysis (break down of glycogen). Glycogen phosphorylase cleaves glycogen to glucose 1-phosphate (first step in glycogenolysis). This pathway mobilizes stored glycogen in liver to replenish used glucose. It also breaks down glycogen in muscle to glucose, to produce energy during physical work. If this enzyme is deficient, then used glucose cannot be replaced leading to hypoglycemia.

In lysosomal glycosidase, there is still normal blood sugar levels (no hypoglycemia). α-amylase and ɣ-amylase is involved in digestion of polysaccharides. Glucose 6-phosphate dehydrogenase (G6PD) is the rate-limiting enzyme in pentose phosphate pathway which catalyzes an irreversible oxidation of glucose 6-phosphate to 6-phosphogluconolactone.

3. During  an operation a patient got injection  of  muscle  relaxant dithylinum. Relaxation of skeletal  muscles and inhibition of respiration lasted  two hours.  This condition was caused  by absence  of the following enzyme in blood serum:

Explanation

4. A couple  has a son with haemophilia. The parents are healthy  but the maternal grandfather also has haemophilia. Specify the type of inheritance:

Explanation

   image

      Recessive because a generation was left out (skipped- grandfather to son); it is sex linked because its from grandfather to son (only the male is involved).

5.

Examination  of  newborns   in  one  of the  Ukrainian cities revealed  a baby  with phenylketonuria. The baby’s parents don’t  suffer  from  this  disease  and  have two    other     healthy    children.    Specify the most likely parents’ genotype with phenylketonuria gene:

Explanation

1.     Aa X Aa

AA  Aa  Aa  aa

AA-healthy (25%)

Aa-carriers (heterozygous; 50%)

aa-sick with hemophilia (25%)

Option B will not produce a sick child.

Option C: all children will be sick.

Option D: 50% of children are carriers; the other 50% have hemophilia

 

Option E: 75% are healthy; 25% has hemophilia

6. A patient has a right-sided  fracture in the region of the frontal third of mandible accompanied by a haematoma in the region of chin. It is caused by the injury of the following artery:

Explanation

krushkrok No6 (2012)

Mental artery is the terminal branch of inferior alveolar artery. It supplies the mental region (chin) and the lower lip.

Inferior labial is a branch of facial artery that supplies the lower lip. Lingual artery supplies the tongue. Facial artery gives branches such as ascending palatine artery, submental artery, superior and inferior labial arteries and angular artery. Palatine artery supplies the palate.

7.

A 46 year old female patient has a continuous history of progressive muscular (Duchenne’s) dystrophy.  Which blood enzyme   changes   will  be   of  diagnostic value in this case?

Explanation

Creatine kinase/Creatine phosphokinase is an enzyme present in striated muscles, used to generate energy. When its serum level is elevated it is often an indication of muscle damage caused by injury, muscular dystrophy or cardiac problems. Most forms of muscular dystrophy are associated with decreased creatinine excretion. Creatinine is a break down product of creatine. Normal constituent of urine but can be elevated in muscular dystrophy. Since creatine is not a normal constituent of urine, it is more specific in this case.

8. As a result  of a cold a patient has the abnormal pain and temperature sensitivity of the frontal  2/3 of his tongue.  Which nerve must have been damaged?

Explanation

image

Lingual nerve is a branch of mandibular nerve (CNV3). Mandibular nerve is the third division of the trigeminal nerve. It’s a sensory nerve that transmits the impulses of pain, temperature and tactile sensitivity. Glossopharyngeal provides taste and tactile sensory innervations to the posterior 1/3 of the tongue.

9.

A 60 year old patient has impaired perception   of   high-frequency   sounds. These changes were caused by damage  of the following auditory analyzer structures:

Explanation

FullSizeRender (25) FullSizeRender (26)

Two membranes divide the spiral canal of cochlea into 3 compartments – vestibular and basilar membrane. Along the basilar membrane are 20,000-30,000 tiny fibers called basilar fibers. Each fiber has different size and shape. Fibers near the oval window (base) are short and stiff (narrowest), while approaching towards helicotrema (apex), the basilar fibers gradually become longer and soft (widest). The motion of the basilar membrane is generally described as a travelling wave. The parameters of the membrane (stiffness, narrow, wide) at a given point along its length determine its characteristic frequency at which it is most sensitive to sound vibrations. High frequency sounds localize near the base of the cochlea (near the round and oval windows), while low frequency sounds localize near the apex (helicotrema); and the middle frequency sounds – middle part of helix. 

There is an impaired perception of high frequency sounds; that means there is damage near the base of cochlea (i.e. near the round and oval windows).

10.

After  a trauma of soft  tissues  in the region  of the  posterior surface  of medial condyle  of humerus  a patient has  got a skin prickle  of medial  forearm surface. Which of the listed nerves is located in the affected region?

Explanation

   \"krushkrok\"krushkrok

     The Ulnar nerve arises from the medial cord of the brachial plexus. It runs along the medial bicipital groove, and proceeds to the ulnar groove situated on the posterior surface of the medial epicondyle of humerus. There the nerve runs covered by fascia and skin only.

Musculocutaneous nerve arises from the lateral cord then traverses the coracobrachialis and appears in between the biceps brachii and the brachialis muscles.

Dorsal scapular nerve (nervus dorsalis scapulae) runs to the levator scapulae and the rhomboid muscles.

Subscapular nerve (nervus subscapularis) supplies the subscapularis and the teres major muscle.

Radial nerve (nervus radialis) arises from the posterior cord of brachial plexus. It passes the radial canal along with deep artery of arm. The nerve quits the canal via its inferior opening (in between the brachialis and the brachioradialis muscle) that leads to the cubital fossa; here at the head of radius, the nerve splits into the superficial and deep branches.

11. A  boy  has  fallen  down  from  a tree. Now he finds it difficult to abduct  his arm into horizontal position. Which muscle is most likely to be injured?

Explanation

    

      Deltoid muscle (M. deltoideus): abducts the shoulder; anterior fibers flex the shoulder, while posterior fibers extend it.

 

Triceps brachii and anconeus extends the forearm; the long head of triceps brachii extends and adducts the shoulder; coracobrachialis muscle flexes and adducts the arm; supinators rotates (supinates) the forearm.

12.

A  female  patient with  a  tumour of pancreas has developed mechanic  jaundice resulting  from  compression of a bile-excreting duct. Which duct is compressed?

Explanation

image

The closest bile excreting duct to the pancreas is the common bile duct (ductus choledochus). So, in case of a tumor affecting the pancreas, this can compress the bile duct, thereby obstructing bile flow leading to mechanic, obstructive or post-hepatic jaundice.

13.

A 28 year old woman  has  been  diagnosed with extrauterine pregnancy complicated by the fallopian tube rupture. The blood is most likely to penetrate the following peritoneal space:

Explanation

krushkrok N013 (2012)

     In the female lesser pelvis, there are two excavations: the rectouterine pouch and vesicouterine pouch. The rectouterine pouch (pouch of douglas) is an intraperitoneal space between the uterus and the rectum. It is a common place for pelvic fluid or blood from hemorrhage to collect after surgery, or rupture of any etiology.

The vesicouterine pouch is between the urinary bladder anteriorly and the uterus posteriorly. It is a shallower recess (pouch).

14.

Patients with similar complaints  applied to the doctor: weakness, pain in the intestines, disorder of GIT.  Examination of the faeces revealed that one patient with four nucleus  cysts should  be hospitalized  immediately.   For what  protozoa are such cysts typical?

Explanation

        krushkrok No14 (2012)krushkrok No14a (2012)

     Entamoeba histolytica is a protozoa causing gastrointestinal infections, which can lead to amebic dysentery or liver abscess. It is transmitted by its cysts in water. It is diagnosed by stool samples or serological methods: trophozoites (with RBCs in the cytoplasm) or cysts (with up to 4 nuclei) in stool.

15.

Autopsy of a 58 year old man revealed that bicuspid valve was deformed, thickened   and   unclosed.    Microscopically: foci of collagen   fibrilla  are   eosinophilic, react positively to fibrin. The most probably it is:

Explanation

1.     Due to the positive fibrin reaction and eosinophilia of the collagen fibers, it is better to choose fibrinoid swelling since there is no record of any inflammatory reaction or cells. Fibrinoid swelling changes is a stromal vascular proteinous degeneration. The main signs are revealed microscopically: the bands of collagen fibers are homogenous, impregnated with plasma proteins.

16.

Chronic inflammation and transformation   of   the   one-layer ciliated   epithelium  into   multiple-layers  flat  epithelium was  revealed   in  the  thickened  mucous membrane  of  the  bronchus bioptate  of the  patient with smoke  abuse.  Which  of the processes is the most likely?

Explanation

IMG_9796

Metaplasia is a reversible change of one type to another type of adult epithelial or mesenchymal cells, usually in response to abnormal stimuli and often reverts back to normal on removal of stimulus. Types (2):

·        Epithelial: Squamous metaplasia: in bronchus (chronic smokers), cervix

  Columnar metaplasia: there is transformation to columnar epithelium.

·        Mesenchymal metaplasia: osseous and cartilaginous.

Hyperplasia: ↑number of cells; Hypertrophy: ↑in size.

\"IMG_9796\"

17.

During  postembryonal haemopoiesis in the  red  bone  marrow  the  cells of one of  the  cellular  differons   demonstrate a gradual  decrease in cytoplasmic  basophilia  as  well  as  an  increase   in  oxyphilia, the nucleus is being forced out. Such morphological changes are typical for the following haemopoiesis type:

Explanation

    Proerythroblast → basophilic erythroblast → polychromatophilic erythroblast → orthochromatophilic erythroblast (normoblast) → polychromatophilic erythrocyte (reticulocyte) → Erythrocyte (red blood cell).

 

The orthochromatophilic erythroblast (normoblast) has a small, compact, densely stained nucleus. The cytoplasm is eosinophilic because of the large amount of hemoglobin. It is only slightly larger than a mature erythrocyte. It loses its nucleus  by extruding it from the cell, it is then ready to pass into a blood sinus of the red bone marrow.

IMG_9922

18. Students who are taking examinations often have dry mouth.   The mechanism that causes this state is the realization of the following reflexes:

Explanation

Reflex activity is the response to a peripheral nervous stimulation that occurs without our consciousness. It can be:

* Inborn or unconditioned: present from birth, does not require previous learning or training.

 

* Acquired or conditioned: developed after conditioning or training. Acquired after birth.

It is conditioned because the students have had a previous experience with examinations before. The manifestation of dry mouth is a sympathetic effect.

19. The temperature of the ambient environment is 38oC and relative air humidity is 50%. What ways of heat emission provide maintaining a constant temperature of the human body?

Explanation

Heat Radiation is a way the surface of the human body emits heat to the environment in the form of infrared rays. The amount of heat the body radiates to the environment is proportional to the surface of radiation area and to the difference between the mean values of skin and environment temperature. The surface radiation area is the total surface area of body parts that contact the air. Elimination of heat by radiation increases with a decrease in ambient temperature and decreases with its increase. It is possible to reduce elimination of heat by radiation via reduction of the surface of radiation area (“winding oneself into a ball”). Heat radiation does not require a medium for transfer of heat. (Key words: naked or lightly clothed).

Convection is a way the body eliminates heat by means of transferring heat via moving particles of air or water. To dissipate heat by means of convection, body surface shall be airflowed at a temperature that is lower than the temperature of the skin. At that, air layer contacting with the skin warms up, decreases its density, rises and is replaced by cooler, denser air. By increasing the speed of the air flow (wind, ventilation) heat emission increases significantly as well (forced convection). Convection requires convection current; current of gases or liquids (Key words: air over exposed area of skin).

Evaporation is a way the body dissipates heat to the environment by its evaporation via sweat or evaporation of moisture from the skin and respiratory tract mucous membranes of (“wet” heat loss). Evaporation closely related to relative humidity.

 

Conduction is a way the body eliminates heat by means of direct contact with another object. Heat is transferred down the temperature gradient (i.e. from the object of higher temperature to the object of lower temperature). Conduction requires contact with another object (Key words:  in water). 

20.

A microspecimen of heart shows rectangular cells from 50  to  120mcm large with central position of nucleus, developed myofibrils. The cells are connected by intercalated discs.  These cells are responsible for the following function:

Explanation

1.     Cardiac muscle has the same types and arrangement of contractile filaments as skeletal muscle. Cardiac muscle fibers exhibit densely staining cross bands called intercalated disks, that cross the fibers in a linear fashion or frequently in a way that resembles the risers of a stairway. The intercalated disks represent highly specialized attachment sites between adjacent cells. The cardiac muscle nucleus lies in the center of the cell which is one feature that helps distinguish them from multinucleated skeletal muscle fibers, whose nuclei lie immediately under the plasma membrane. Cardiac muscle cells exhibit a spontaneous rhythmic contraction.

21. A 30 year old woman has subnormal concentration of  enzymes  in  the pancreatic juice. This might be caused by the hyposecretion of the following gastrointestinal hormone:

Explanation

·        Cholecystokinin-pancreozymin: source – I cells in duodenum and jejunum of small intestine. Action – increase pancreatic secretion, gallbladder contraction, sphincter of oddi relaxation and decrease gastric emptying. It acts on neural muscarinic pathways to cause pancreatic secretion.

·        Somastostatin: source – D cells (pancreatic islets, gastrointestinal mucosa). Action – decrease gastric acid and pepsinogen secretion; decrease pancreatic and small intestine fluid secretion; decrease gallbladder contraction; decrease insulin and glucagon release.

·        Secretin: source – S cells (duodenum). Action – increase pancreatic HCO3- secretion ( ↑ bicarbonate secretion); increase bile secretion; decrease gastric acid secretion.

·        GIP: source - K cells in duodenum and jejunum. Action – exocrine action: decrease gastric hydrogen ion (H+) secretion; endocrine action: increase insulin release. It is also known as Glucose-dependent insulinotropic peptide.

·        VIP: parasympathetic ganglia in sphincters, gallbladder, small intestine. Action – increase intestinal water and electrolyte secretion, relaxation of intestinal smooth muscle and sphincters.

 

Only cholecystokinin-pancreozymin increase pancreatic secretion. Secretin only increases pancreatic HCO3- secretion which neutralizes gastric acid in duodenum, allowing pancreatic enzymes to function.

22.

A    female    patient   consulted     a physician about   digestive disorder, extended abdominal pain. Examination revealed drastic decrease in hemoglobin concentration. It is known from the anamnesis that while living in the Far East the patient used to eat freshly-salted caviar. Some relatives  living with her had the similar condition. What  is the most likely diagnosis?

Explanation

·        Diphyllobothrium latum causes diphyllobothriasis. A fish tapeworm. In contrast to the other cestodes, which have suckers , the scolex of D. latum has two elongated sucking grooves by which the worm attaches to the intestinal wall. Infection by D. latum causes little damage in the small intestine. In some individuals, megaloblastic anemia occurs as a result of vitamin B12 deficiency caused by preferential uptake of the vitamin by the worm. Transmission: ingestion of larvae from raw freshwater fish. Caviar is prepared from fish.

·        Echinococcus granulosus causes Echinococcosis. It is composed of a scolex and only 3 proglottids, making it one of the smallest tapeworms. The scolex has a circle of hooks and 4 suckers similar to Taenia solium. Dogs are the most important definitive hosts. The intermediate hosts are usually sheep. Humans are almost always dead-end intermediate hosts. Transmission: ingestion of eggs from dog faeces. Disease – hydatid cysts in liver causing anaphylaxis if antigens released.

·        Taeniasis: there are two important human pathogens in the genus Taenia: T. solium (pork tapeworm) and T. saginata (beef tapeworm)

·        Trichiniasis (Trichinosis) is caused by Trichinella spiralis (nematode- roundworm). Transmission: fecal-oral; undercooked meat (especially pork). A few days after eating undercooked meat, usually pork, the patient experiences diarrhea followed by 1-2weeks later by fever, muscle pain, periorbital edema and eosinophilia.

 

·        Ascaridiasis (Ascariasis) caused by Ascaris lumbricoides (giant roundworm). The major damage occurs during larval immigration rather than from the presence of the adult worm in the intestines. The principal sites of tissue reaction are the lungs, where inflammation with an eosinophilic exudates occurs in response to larval antigens. Ascaris pneumonia with fever, cough and eosinophilia can occur with a heavy larval burden.

23.

A patient has a trauma of sternocleidomastoid muscle. This caused a decrease in value   of  the   following   indicator   of external  respiration:

Explanation

image

All muscles that elevate the rib cage are muscles of inspiration and those that depress the rib cage are muscles of expiration.

Muscles of inspiration:

·        Sternocleidomastoid: lift upward on the sternum.

·        Anterior serrati: lift many of the ribs.

·        Scalene: lift the first two ribs.

Muscles of expiration: Abdominal recti – pull down the lower ribs and other abdominal muscles also compress the abdominal contents upwards against the diaphragm and internal intercostals.

Normal respiration is accomplished by the movement of the diaphragm only.

24. A month   after surgical constriction of rabbit’s renal artery the considerable increase   of  systematic   arterial  pressure was observed.  What of the following regulation mechanisms   caused   the animal’s pressure change?

Explanation

FullSizeRender (12)

↓blood supply → activates renin. Renin catalyzes the conversion of angiotensinogen to angiotensin I. Angiotensin converting enzyme (ACE) converts angiotensin I → angiotensin II. Angiotensin II causes:

- vasoconstriction → ↑BP

- vasopressin (ADH) → ↑H2O reabsorption → ↑plasma volume

- aldosterone → ↑Na+ and H2O reabsorption → ↑plasma volume → ↑BP

\"FullSizeRender

25.

Microscopic examination of the enlarged  neck   gland   of  a  14  year   old girl   revealed    destruction   of    the    tissue  structure  of  the  node,   absence   of the lymph follicles, sclerotic areas and necrosis foci, cell constitution of the node is polymorphous, lymphocytes,  eosinophils, big atypical cells with multilobular nuclei (Beresovsky-Sternberg cells) and mononuclear cells  of  the  large  size  are present. What is the most likely diagnosis?

Explanation

FullSizeRender (28)
26. A  child  has  abnormal  formation  of tooth  enamel and dentin  as a result of low concentration of  calcium  ions  in  blood. Such abnormalities might be caused  by deficiency of the following hormone:

Explanation

image Parathyroid hormone: secreted by chief cells of parathyroid gland. Effects include:

↑bone resorption of Ca2+ and PO43- → ↑their plasma levels

↑kidney reabsorption of Ca2+ in distal convoluted tubule → ↑ Ca2+ plasma level

↓reabsorption of PO43- in proximal convoluted tubule → ↓ PO43- plasma levels

↑Calcitriol (vit D3) production by stimulating kidney 1α-hydroxylase in proximal convoluted tubule. It increases Ca2+ and PO43- absorption in the intestine.

In general, parathyroid hormone ↑ Ca2+ plasma level but ↓ PO43- plasma levels. Abnormal synthesis (↑synthesis) of parathyroid hormone can lead to hypercalcemia and hypophosphatemia.

Calcitonin is secreted by parafollicular cells (C cells of the thyroid gland). It ↓bone resorption of Ca2+. It opposes actions of parathyroid hormone. But its not important in normal Ca2+ homeostasis. Calcitriol ↑ circulating Ca2+ ions as a means of enhancing intestinal absorption of calcium (NB: Calcitriol production is dependent on parathyroid hormone). Aldosterone: reabsorption of Na+, excretion of K+. Cortisol – glucocorticoid: ↑ blood pressure and gluconeogenesis; ↓ inflammatory and immune responses; ↓ bone formation (↓osteoblast activity).

Likewise, deficiency of parathormone can lead to low Ca2+ levels.

27.

A sportsman was examined after an intensive physical activity. The examination revealed disorder of movement coordination but the force of muscle  contractions remained the same. It can be explained by retarded speed of excitement conduction through:

Explanation

   One of the properties of synapse is fatigue. During continuous muscular activity, synapse becomes the seat of fatigue along with Betz cells present in motor area of frontal lobe of cerebral cortex. Fatigue at synapse is due to the depletion of neurotransmitter substance, acetylcholine. Depletion of acetylcholine occurs because of 2 factors:

·        Soon after the action, acetylcholine is destroyed by acetylcholinesterase.

·        Due to continuous action, new acetylcholine is not synthesized.

The excitatory terminals on many neurons can store enough excitatory transmitter substance to cause only about 10,000 action potentials and the neurotransmitter can be exhausted in only a few seconds to a few minutes of rapid stimulation.

            It is fatigue of central synapses because only the function of muscle   coordination is affected which is an exclusive function of the higher brain centers. Force of contraction s determined by discharge at neuromuscular synapses in the neuromuscular junction.

28. After     a    long    training     session a sportsman has developed fatigue accompanied by abrupt performance decrement. What link  of the  reflex  arch was the fatigue initiated  in?

Explanation

Reflex arch is the anatomical nervous pathway for a reflex action. A simple reflex arc has 5 components:

·        Receptor: end organ, which receives the stimulus.

·        Afferent nerve or sensory nerve: transmits sensory impulses from the receptor to center.

·        Center: receives the sensory impulses via afferent nerve fibers and in turn, it generates appropriate motor impulses. Center is located in the brain or spinal cord.

·        Efferent nerve or motor nerve: transmits motor impulses from the center to the effector organ.

·        Effector organ: is the structure such as muscle or gland where the activity occurs in response to stimulus.

When a reflex activity is continuously elicited for a long time (long training session), the response is reduced slowly and at one stage, the response does not occur. This stage of failure to give response to the stimulus is called fatigue. Center or the synapse of the reflex arc is the first seat of fatigue.

29. Blood minute volume of a 30 year old woman at rest is 5 l/m. What blood volume is pumped through the pulmonary vessels per minute?

Explanation

Blood minute volume is the amount of blood pumped out by each ventricle in one minute. It is the product of stroke volume and heart rate. Normal value is 5L/ventricle/minute.

 

If blood minute volume is 5L/m; the volume pumped through the pulmonary vessels (lung vessels) per minute by the right ventricle equals 5L.

30.

After    a   tourniquet application  a patient was found  to have petechial haemorrhages. The reason for it is the dysfunction of the following cells:

Explanation

1.     Thrombocyte (platelet) is involved in primary hemostasis. It’s a small cytoplasmic fragment derived from megakaryocytes. It has a life span of 8-10days. When activated by endothelial injury, it aggregates with other platelets and interacts with fibrinogen to form platelet plug. Thrombocytopenia or decrease platelet function/dysfunction results in petechiae.

 

Leukocytes: Neutrophils (54-62%); Lymphocytes (25-33%); Monocytes (3-7%); Eosinophils (1-3%); Basophils (0-0.75%). Neutrophils, eosinophils, basophils are granulocytes; while lymphocytes, monocytes are mononuclear cells and they are also agranulocytes. They are all responsible for defense against infections.

31.

A patient under test was subjected to a moderate physical stress. His minute blood volume amounted 10 l/min.  What blood volume was pumped   through his lung vessels every minute?

Explanation

Blood minute volume is the amount of blood pumped out by each ventricle in one minute. It is the product of stroke volume and heart rate. Normal value is 5L/ventricle/minute.

If blood minute volume is 5L/m; the volume pumped through the pulmonary vessels (lung vessels) per minute by the right ventricle equals 5L.

 

Same for any value given.

32.

A patient presents with the following motor activity disturbances: tremor, ataxia and asynergia movements, dysarthria. The disturbances are most likely to be localized in:

Explanation

33.

A  man  has  a  considerable decrease in diuresis  as a result  of 1,5 l blood  loss. The primary   cause of  such  diuresis  disorder is the hypersecretion of the following hormone:

Explanation

1.     Antidiuretic hormone (vasopressin) is secreted in response to decrease blood volume and increase plasma osmolarity. It binds to receptors on principal cells of collecting ductules causing increase number of aquaporins and increase water reabsorption which leads to decreased diuresis.

Atrial natriuretic peptide is secreted in response to increase atrial pressure. It causes increase glomerular filtration rate (GFR) and increase sodium ion filtration with no compensatory sodium ion reabsorption and water in distal nephron which leads to increase diuresis.

 

Parathyroid hormone is secreted in response to decrease plasma calcium ion, increase phosphate ion or decrease plasma calcitriol [1,25-(OH)2D3].

34.

The  patient with  thymoma (thymus gland  tumour) has cyanosis, extension of subcutaneous venous  net  and  edema  of the  soft tissues  of face, neck,  upper  part of the trunk  and upper  extremities. What venous trunk is pressed with tumour?

Explanation

1.     The superior vena cava (SVC) arises from merging left and right brachiocephalic veins posterior to the right first sternocostal joint. The SVC descends to enter the right atrium. On the left, one can distinguish the ascending aorta and on the right – the mediastinal pleura and phrenic nerve. The right pulmonary vein resides posterior to the SVC & the thymus and the right lung – anterior to it. Therefore, thymoma (thymus gland tumor) will definitely press on the SVC. The brachiocephalic veins drain the head, neck and upper limbs; hence, the manifestations.

35.

An experimental animal that was kept on  protein-free  diet  developed  fatty  liver  infiltration,   in  particular as  a  result of deficiency  of methylating agents.  This is caused by disturbed generation of the following metabolite:

Explanation

krushkrok No17 (2014)Hepatic steatosis can occur when humans are deprived of choline.

Choline + Phosphatidic acid → Phosphatidylcholine (lecithin, PC). In the liver PC can also be synthesized from phosphatidylserine (PS) and phosphatidylethanolamine (PE), when free choline levels are low, because it exports significant amounts of PC in bile and as a component of serum lipoproteins (needed for fat metabolism)

PS → PE →→→ PC. 3 methylation reactions between PE and PC. S-adenosylmethionine is the methyl group donor. If choline, phosphatidylcholine or methionine is deficient, there will be abnormal phospholipid synthesis, oxidative damage caused by mitochondrial dysfunction, lipoprotein secretion (remember, if VLDL cannot be secreted it will be accumulated & cause fatty liver degeneration as seen in hepatic steatosis). PC is also a major lipid component of lung surfactant.

36.

Vegetative abnormalities in the sleep, heat  regulation, all kinds  of metabolism, diabetes insipidus  are  developing  in the patient due  to  growth  of  the  tumour in the III ventricle  of brain. Irritation of the nucleus of what part of the brain can cause this symptoms?

Explanation

 Hypothalamus is a diencephalic structure. Diencephalon: thalamus, hypothalamus, pineal body, third (3rd) ventricle. The Hypothalamus is situated just below thalamus in the ventral part of diencephalon. It is the important part of brain, concerned with homeostasis of the body. It regulates endocrine functions, metabolic activities, hunger, thirst, sleep, wakefulness, emotion, sexual functions. Lesion of hypothalamus can occur due to tumors, encephalitis and ischemia; accompanied by disturbances in carbohydrate and fat metabolisms; disturbance in sleep and sexual functions. It can also result in some clinical manifestations such as: Diabetes insipidus, dystrophia adiposogenitalis, kallmann syndrome, Laurence-Moon-Biedl syndrome, narcolepsy, cataplexy.

37.

A patient complains   about   dyspnea provoked by the physical activity.  Clinical examination revealed anaemia and presence of the paraprotein in the zone of gamma-globulins. To confirm the myeloma diagnosis it is necessary to determine the following index in the patient’s urine:

Explanation

   Paraprotein, myeloma protein, M protein or spike protein is an abnormal immunoglobulin (Ig) fragment or immunoglobulin (Ig) light chain that is produced in excess by an abnormal clonal proliferation of plasma cells, typically in multiple myeloma. Monoclonal free light chains in the serum or urine are called bence jones (BJ) proteins.

Bence jones (BJ) protein: free kappa (κ) or lambda (λ) light chains that are excreted in urine associated with plasma cell malignancies (myeloma) and Waldenstrὄm macroglobulinemia. In myeloma, urinalysis for BJ protein is positive in 60-80% of cases.

38.

Examination  of  a  child  who  hasn’t got fresh  fruit  and  vegetables  during  winter revealed numerous subcutaneous hemorrhages, gingivitis, carious cavities in teeth.  What vitamin combination should be prescribed in this case?

Explanation

Vitamin P (rutin) – permeability vitamin. They reduce permeability of blood vessels, especially capillaries. It prevents hyaluronic acid (the basic compound of the extracellular matrix) from degeneration by inhibition of the enzyme hyaluronidase. Ascorutin is a drug containing vitamin C and P; it is used to decrease the permeability of blood vessels.

Vitamin C (ascorbic acid): found in fruits and vegetables; an antioxidant; also facilitates iron absorption by reducing it to Fe2+ state. It is necessary for hydroxylation of proline and lysine in collagen synthesis; necessary for dopamine β-hydroxylase, which converts dopamine to norepinephrine. Deficiency leads to: scurvy – swollen gums, bruising, petechiae, hemarthrosis, anemia, poor wound healing, perifollicular and subperiosteal hemorrhages, “corkscrew” hair; Weakened immune response.

Type III collagen is found in blood vessels; Type IV collagen is found in basement membrane. Deficiency in Vitamin C disrupts the second stage of collagen synthesis in fibroblasts (hydroxylation of collagen) which results in petechiae, bruising, hemarthrosis.

Vitamin B2 (riboflavin) deficiency – growth retardation, glossitis, conjunctivitis

Vitamin B1 (thiamine) deficiency – Beri-Beri (polyneuritis)

Vitamin A (retinol) deficiency – Night blindness

Vitamin B9 (folic acid) deficiency – macrocytic megaloblastic anemia

39.

A   2 year   old   child   with   mental and physical retardation has been delivered to a hospital. He presents with frequent vomiting   after   having   meals. There is phenylpyruvic acid in urine. Which metabolism abnormality is the  reason for this pathology?

Explanation

   Phenylketones – phenylacetate, phenyllactate and phenylpyruvate. These compounds are not normally produced in significant amounts in the presence of functional phenylalanine hydroxylase, but are elevated in phenylketonuria. These metabolites give urine a characteristic musty (“mousey”) odour. The disease acquired its name from the presence of a phenylketone (now known to be phenylpyruvate) in the urine.

Phenylketonuria (PKU) is caused by a deficiency of phenylalanine hydroxylase. PKU is the most common clinically encountered inborn error of amino-acid metabolism. Biochemically, it is characterized by accumulation of phenylalanine and a deficiency of tyrosine. It may also be caused by deficiencies in tetrahydrobiopterin cofactor (BH4) or in dihydropteridine (BH2) reductase, which regenerates BH4 from BH2.

40. 46 chromosomes were revealed on karyotype examination of the  5 year  old girl. One of the 15th pair of chromosomes is longer than usual due to connected chromosome from the 21 pair. What type of mutation does this girl have?

Explanation

FullSizeRender (29)1.     Translocations occur when two chromosome regions join together, when they would not normally. Chromosome translocations in somatic cells may be associated with tumorogenesis. We can have reciprocal and robertsonian translocations. Examples of diseases due to translocations are: Burkitt lymphoma t(8:14); Chronic myelogenous leukemia t(9;22) – Philadelphia chromosome.

Duplications occur when a portion of the chromosome is present on the chromosome in two copies.

Inversion involve an end-to-end reversal of a segment within the same chromosome (turn of 180o).

Deletion is a mutation in which a part of a chromosome or a sequence of DNA is missing. Deletion is the loss of genetic material.

41.

A   62   year   old   woman   complains       of   frequent   pain    attacks    in   the area   of   her   chest   and   backbone, rib fractures. Her doctor suspected myeloma (plasmocytoma). What of the following laboratory characteristics will be of the greatest diagnostic importance?

Explanation

Myeloma/Plasmocytoma/Multiple Myeloma

1.     Paraprotein, myeloma protein, M protein or spike protein is an abnormal immunoglobulin (Ig) fragment or immunoglobulin (Ig) light chain that is produced in excess by an abnormal clonal proliferation of plasma cells, typically in multiple myeloma. Monoclonal free light chains in the serum or urine are called bence jones (BJ) proteins.

Bence jones (BJ) protein: free kappa (κ) or lambda (λ) light chains that are excreted in urine associated with plasma cell malignancies (myeloma) and Waldenstrὄm macroglobulinemia. In myeloma, urinalysis for BJ protein is positive in 60-80% of cases.

42. Before the cells can utilize the glucose, it is first transported from the extracellular space  through the  plasmatic   membrane inside them. This process is stimulated by the following hormone:

Explanation

In the liver and muscle, insulin increases glycogen synthesis. In the muscle and adipose tissue, insulin increases glucose uptake by increasing the number of glucose transporters (GLUT-2) in the cell membrane. Insulin is an anabolic hormone; it increases glucose uptake, glycogen synthesis, protein synthesis, fat synthesis; and decrease gluconeogenesis, glycogenolysis, lipolysis. Gluconeogenesis is the production of glucose from non-carbohydrate sources.

 

Glucagon is a functional antagonist of insulin.

43.

Parodontitis is treated with  calcium preparations and a hormone that stimulates  tooth  mineralization and inhibits tissue resorption. What hormone is it?

Explanation

image Calcitonin is secreted by parafollicular cells (C cells of the thyroid gland). It ↓bone resorption of Ca2+. It opposes actions of parathyroid hormone. But its not important in normal Ca2+ homeostasis. Calcitriol ↑ circulating Ca2+ ions as a means of enhancing intestinal absorption of calcium (NB: Calcitriol production is dependent on parathyroid hormone).

Parathyroid hormone: secreted by chief cells of parathyroid gland. Effects include:

↑bone resorption of Ca2+ and PO43- → ↑their plasma levels

↑kidney reabsorption of Ca2+ in distal convoluted tubule → ↑ Ca2+ plasma level

↓reabsorption of PO43- in proximal convoluted tubule → ↓ PO43- plasma levels

↑Calcitriol (vit D3) production by stimulating kidney 1α-hydroxylase in proximal convoluted tubule. It increases Ca2+ and PO43- absorption in the intestine.

In general, parathyroid hormone ↑ Ca2+ plasma level but ↓ PO43- plasma levels. Abnormal synthesis (↑synthesis) of parathyroid hormone can lead to hypercalcemia and hypophosphatemia.

44.

A child has an  acute   renal  failure. What biochemical factor found in saliva can confirm this diagnosis?

Explanation

       Kidneys excrete the unwanted waste products which are formed during metabolic activities e.g.

·        Urea: end product of amino acid metabolism

·        Uric acid: end product of nucleic acid metabolism

·        Creatinine: end product of metabolism in muscles

·        Bilirubin: end product of hemoglobin degradation

·        Products of metabolism of other substances.

 

Many substances both organic and inorganic are excreted in saliva also. In some pathological conditions, saliva excretes certain substances which are not found in saliva under normal conditions e.g. glucose in Diabetes mellitus. In certain conditions, some of the normal constituents of saliva are excreted in large quantities e.g. excess urea is excreted in saliva during renal failure, nephritis and excess calcium is excreted during hyperparathyroidism.

45. After implantation of a cardiac valve a young man constantly takes  indirect  anti-coagulants.  His state was complicated by hemorrhage, What substance content has decreased in blood?

Explanation

image          

 Indirect anticoagulants:

·        Coumarine derivatives – Neodicumarin, warfarin, syncumar

·        Indandione derivatives – Phenylinum

They antagonize the cofactor functions of vitamin K. Vitamin K is essential for the formation of various clotting factors in the liver, namely clotting factor II, VII, IX and X; Protein C and S.

Factor II – Prothrombin

Factor VII – Stable factor

Factor IX – Christmas factor

Factor X – Stuart-Prower factor

These factors undergo vitamin K-dependent post-translational modification, whereby a number of their glutamic acid residues are carboxylated to form ɣ-carboxyglutamic acid residues. The ɣ-carboxyglutamyl residues bind calcium ions which are essential for interaction between the coagulation factors and platelet membranes. This oral anticoagulants block epoxide reductase and creation of active form of vitamin K resulting in disturbances in prothrombin and proconvertin synthesis in liver.

All other options are not clotting factors.

46. After severe viral hepatitis a 4 year old boy presents with vomiting,  occasional loss of consciousness, convulsions. Blood test revealed hyperammoniemia. Such condition is caused by a disorder of the following biochemical hepatic process:

Explanation

image   

Hyperammonemia can be acquired (e.g. liver disease) or hereditary (e.g. urea cycle enzyme deficiencies). Acquired hyperammonemia may be a result of an acute process e.g. viral hepatitis, ischemia or hepatotoxins. As a result, the detoxification of ammonia (i.e. its conversion to urea) is severely impaired, leading to elevated levels of circulating ammonia.

47.

Having  helped  to  eliminate consequences  of  a  failure   at  a  nuclear power  plant,  a  worker  got  an  irradiation  doze  of  500  roentgen.  He  complains of  headache, nausea,  dizziness.  What changes in leukocytes quantity can be expected 10 hours after irradiation?

Explanation

Neutrophils are the first leukocytes that cross the blood vessel wall to enter inflammatory sites. Under normal conditions, leukocytes are restricted to the center of small blood vessels, where the flow is fastest. In inflammatory sites, where the vessels are dilated, the slower blood flow allows the leukocytes to move out of the center of the blood vessel and interact with the vascular endothelium. Even in the absence of infection, monocytes migrate continuously into the tissues, where they differentiate into macrophages; meanwhile, during an inflammatory response, the induction of adhesion molecules on the endothelial cells, as well as induced changes in the adhesion molecules expressed on leukocytes recruit large numbers of circulating leukocytes, initially Neutrophils and later monocytes, into the site of an infection (inflammatory focus).

 

First –Neutrophils; second –monocytes and macrophages; third –lymphocytes.

The nuclear radiation this patient was exposed to can produce inflammatory reaction in the patient. Within the first 24hours, the first leukocyte to get into any inflammatory focus are the neutrophils, followed by the monocytes (macrophages) and then lymphocytes.

48.

Bacterioscopic examination of a smear from the  pharynx  of a diphtheria suspect revealed   bacilli  with  volutine   granules. What etiotropic drug  should  be  chosen in this case?

Explanation

1.     Etiotropic drug deals directly with the etiological factor (diphtheria toxin). Antidiphtheric antitoxic serum neutralizes the toxin of diphtheria and is employed both as a curative and as a prophylactic agent. Diphtheric anatoxin (diphtheria toxoid) is the diphtheria toxin that has been weakened until it is no longer toxic but is strong enough to induce the formation of antibodies and immunity to the disease.

49.

A    patient    with    a    history     of chronic      glomerulonephritis     presents with azotemia, oliguria, hypo- and isosthenuria, proteinuria. What is the leading factor in the pathogenesis of these symptoms   development under    chronic renal failure?

 

Explanation

Chronic renal failure is the progressive, long standing and irreversible impairment of renal functions. When some of the nephrons loose their function, the unaffected nephrons can compensate for it. However, when more and more nephrons start losing their function over the months and years, then the compensatory mechanism fails and chronic renal failure manifests.

50.

After an attack  of bronchial  asthma  a patient had  his  peripheral blood  tested. What changes can be expected?

Explanation

Type I (Immediate, Anaphylaxis, Reagin): IgE (immunoglobulin E)-dependent activation of mast cells/basophils, usually accompanied by eosinophilia e.g. urticaria (hives), hay fever, asthma (wheezing), rhinitis and conjunctivitis (stuffy nose and itchy eyes; usually seasonal)

This is an example of type I hypersensitivity reaction with the expression of eosinophilia.

51.

A 70 year old female patient was diagnosed  with fracture of left femoral neck accompanied by disruption of ligament  of head  of femur. The branch of the following artery is damaged:

Explanation

  krushkrok No51a (2012)No1 is the Obturator artery...

   Obturator artery reaches the thigh region via the obturator canal. The artery supplies the adductors of thigh and the hip joint. The hip joint receives blood from the acetabular branch, which passes within the ligament of head of femur. The obturator artery anastomoses with the inferior hypogastric artery via the pubic branch.

Femoral artery arises directly from the external iliac artery. On leaving the vascular space, it appears within the femoral triangle together with the femoral nerve (laterally) and femoral vein (medially).

The external iliac artery descends on the medial aspect of psoas major muscle and quits the lesser pelvis via the vascular space to become continuous with the femoral artery. Common iliac arteries gives both external and internal iliac arteries.

Inferior gluteal artery leaves the lesser pelvis cavity via the infrapiriform foramen. Within the gluteal region, it supplies the gluteus maximus and other related muscles.

Internal pudendal artery passes inferolaterally, anterior to the piriformis muscle and sacral plexus. It leaves the pelvis between the piriformis and the coccygeus muscles by passing through the inferior part of the greater sciatic foramen.

52.

ECG of a patient shows prolongation of T-wave. This is caused by deceleration in ventricles of:

Explanation

krushkrok No28a (2009) krushkrok No28 (2009) ‘T’ wave is the final  ventricular complex and is a positive wave. It is due to the repolarization of ventricular musculature. Normal duration is 0.2sec; normal amplitude is 0.3mV.
53.

After  the  traumatic tooth  extraction a  patient is  complaining   of  acute,  dull, poorly-localized pain in gingiva, body temperature rise up to 37, 5oC . The patient has been diagnosed with alveolitis. Specify the kind of pain in this patient:

Explanation

Protopathic pain: poorly localized pain

* Radiating pain: perceived at a site adjacent to or away from the site of origin but in the same dermatome i.e. supplied by afferent nerve fibers of one dorsal root.

* Viscera pain: organs, poorly localized, diffuse

* Epicritic pain: well localized pain

* Phantom pain: pain from non-existent body structures (amputated limbs)

 

Phantom – if a leg has been amputated, the cut end heals with scar formation. The cut ends of nerve fibers are merged within the scar. If the cut end of sensory fibers are stimulated during movement of thigh, the patient feels as if the sensation is originating from non-existent leg. Sometimes the patient feels pain in non-existent limb.

54.

A 50  year  old  patient suffers  from essential   hypertension. After   a physical stress he  experienced muscle  weakness, breathlessness, cyanosis  of lips, skin and face. Respiration was accompanied by distinctly heard bubbling rales. What mechanism underlies the development of this syndrome?

Explanation

FullSizeRender (30)   Acute heart failure refers to sudden and rapid onset of signs and symptoms of abnormal heart functions. Chronic heart failure is characterized by the symptoms that appear slowly over a period of time and become worst gradually.

The main symptoms of right-sided heart failure are fluid accumulation and swelling (edema) in the feet, ankles, legs, liver and abdomen. Left-sided heart failure leads to fluid accumulation in the lungs, which causes shortness of breath. At first, shortness of breath occurs only during exertion, but as heart failure progresses, it occurs with less and less exertion and eventually occurs even at rest. Moist and dry rales are heard over the lungs.

After a physical stress – sudden and rapid – ACUTE

Rales are caused by congestion of the lungs – left ventricular heart failure.

55. A 5 year old child has been diagnosed with    acute     right    distal    pneumonia. Sputum inoculation revealed that the causative agent  is resistant  to  penicillin, but it is sensitive to macrolides.  What drug should be prescribed?

Explanation

      Macrolides inhibit protein synthesis by blocking translocation, they bind to the 23S rRNA of the 50S ribosomal subunit. It is bacteriostatic. Examples of macrolides includes azithromycin, clarithromycin, erythromycin etc.

All other options are not macrolides, but are antibiotics.

Ampicillin is an extended spectrum penicillin; a β-lactam antibiotic.

 

Gentamycin and streptomycin are Aminoglycosides. Together with tetracyclines, they are 30S ribosomal subunit inhibitor.

56.

A patient suffering from chronic myeloleukemia   has    got    the    following symptoms  of  anemia:  decreased number  of erythrocytes and low haemoglobin    concentration,    oxyphilic and  polychromatophilic normocytes, microcytes. What is the leading pathogenetic mechanism of anemia  development?

Explanation

Chronic myeloleukemia is characterized by extensive production of blast (immature) cells in the blood. The immature cells (e.g. immature RBCs) cannot perform the normal function of a mature cell, hence the manifestation of anemia. The many blast cells constitute the majority of cells found in blood during leukemia. Therefore, as soon as cells die they are replaced by the immature ones, which are usually overproduced in leukemia.

57.

A patient with obliterating endarteritis underwent ganglionic sympathectomy. What type of arterial hyperaemia should have developed as a result of the surgery?

Explanation

·        Neuroparalytic arterial hyperemia is caused by damage or blockage of α-adrenoreceptors (sympathetic nervous system). It is characterized by reduction or absence (paralysis) of the sympathetic nervous system effects on the walls of the arteries and arterioles.

 

·        Neurotonic arterial hyperemia is caused by activation of parasympathetic nervous system; irritation of vascular dilators part of vascular center (CNS) or inhibition of vascular – constrictor part of this center (vasomotor center of CNS); M-cholinoreceptors; H2-histaminereceptors. It is characterized by predominance of the parasympathetic nervous system effects on arterial vascular walls.

58.

Autopsy of  a  woman  with  cerebral atherosclerosis  revealed   in  the  left cerebral hemisphere a certain  focus that is  presented by  flabby,  anhistic,  greyish and yellowish tissue with indistinct  edges. What pathological process is the case?

Explanation

      Stroke is sudden loss of blood circulation to an area of the brain resulting in a corresponding loss of neurologic function. Types:

·        Ischemic (70-80% of cases):

- Atherosclerotic (thrombotic): most common type

- Embolic

·        Intracerebral hemorrhage

·        Subarachnoid hemorrhage

·        Lacunar stroke

 

Atherosclerotic (thrombotic) stroke: Ischemic type of stroke is commonly caused by platelet thrombosis that develops over a disrupted atherosclerotic plaque. Characteristics: it usually develops at the periphery of the cerebral cortex, swelling of brain, loss of dermacation between gray and white matter, breakdown of myelin, pale infarct; Cystic area develops after 10days to 3weeks due to liquefactive necrosis.

59. A patient suffering from chronic hyperacidic    gastritis   takes    an   antacid drug for heartburn elimination. After its ingestion the patient feels better but at the same time he has a sensation  of stomach swelling. Which of the following drugs might be the cause of such side effect?

Explanation

     Antacids are weak bases that react with gastric acid to form water and a salt to diminish gastric acidity. Because pepsin is inactive at a pH greater than 4, antacids also reduce pepsin activity. They can be:

·        CO2 forming: sodium hydrocarbonate (or carbonate); calcium carbonate

·        Non-forming CO2: magnesium hydroxide, aluminum hydroxide

Uses: hyperacid gastritis, gastric and duodenal ulcers; poisoning with acids.

 

Adverse effects: in addition to the potential for systemic alkalosis, sodium hydrocarbonate liberates CO2 causing belching and flatulence (sensation of stomach swelling); aluminum hydroxide tends to cause constipation, whereas magnesium hydroxide tends to produce diarrhea.

60. A 63 year  old  patient with  collapse presentations was delivered to the emergency  hospital.  A physician has chosen noradrenalin against hypotension. What is its mechanism of action?

Explanation

   Because norepinephrine is the neuromediator of adrenergic nerves, it should theoretically stimulate all types of adrenergic receptors. In practice, when the drug is given in therapeutic doses to humans, the α-adrenergic receptor is most affected. Norepinephrine causes a rise in peripheral resistance due to intense vasoconstriction of most vascular beds, including the kidney (α1 effect). Both systolic and diastolic blood pressures increase. α1 receptors are present on the postsynaptic membrane of the effector organs and mediate constriction of smooth muscle. NB: norepinephrine (noradrenaline) causes greater vasoconstriction than epinephrine, because it does not induce compensatory vasodilation via β2 receptors on blood vessels supplying skeletal muscles.

61.

Autopsy   of    a    man    who    had tuberculosis revealed a 3x2 cm large cavity in the  superior lobe  of the  right  lung. The cavity was interconnected with a bronchus,   its  wall  was  dense  and  consisted  of  three   layers:  the  internal  layer was pyogenic, the middle layer was made by tuberculous granulation tissue and the external one  was made  by connective tissue. What is the most likely diagnosis?

Explanation

Secondary tuberculosis usually results from reactivation of dormant, endogenous tubercle bacilli in a sensitized patient who has had previous contact with the tubercle bacillus. Reactivation typically begins in the apical or posterior segments (often 1st and 2nd segments) of one or both upper lobes (“simon’s foci”), where the organisms were seeded during the primary infection. There are 8 forms or stages of the secondary tuberculosis: Acute local tuberculosis; Fibrous local tuberculosis; Infiltrative tuberculosis; Tuberculoma; Caseous pneumonia; Acute cavernous tuberculosis; Fibrous cavernous tuberculosis; Cirrhotic tuberculosis.

Fibrous cavernous tuberculosis is the most frequent form. Macroscopically, the lesions are spherical and cavitary (cavity can contain blood and blood clots); the so-called coin lesions. Microscopically, the outer wall of cavity shows fibrosis or sclerosis. Internal surface may be connected with bronchus. The wall of cavern has 3 membranes:

·        Internal membrane occurs by necrotic tissue

·        Middle membrane occurs by special granular tissue

 

·        External membrane occurs by connective fibrous tissue

62. A patient suffering from syphilis has been  treated with  bismuth  preparations. As  a result  of it some  grey spots  turned up on the  mucous  membrane of the  oral cavity; nephropathy symptoms  were  also present. What drug should  be  used  for treatment of bismuth intoxication?

Explanation

Dimercaprol (unithiol, British anti-lewisite) is used to chelate mercury, bismuth, arsenic and in combination with edentate calcium disodium to treat lead intoxication. It contains two sulfuhydryl (SH-) groups and forms two bonds with metal ions. Administered intramuscularly.

 

Mannitol – osmotic diuretic; Nalorphine hydrochloride – antidote, blocks opoid receptors and replace opoids from binding with them. Metamizole: analgesic, non-opoid drug. Calcium chloride – Calcium preparation, coagulant with systemic action.

63.

Autopsy of a man who died from ethylene glycol poisoning   revealed   that his   kidneys   are   a   little   bit   enlarged, edematic; their capsule can be easily removed.  Cortical substance is broad and light-grey.  Medullary substance   is dark- red. What pathology had this man?

Explanation

   Acute tubular necrosis/Necrotic nephrosis/Necronephrosis involves the death of tubular epithelial cells that form the renal tubules of the kidneys. Most common cause of acute renal failure. It can be ischemic or nephrotoxic.

·        Ischemic acute tubular necrosis occurs due to hypoperfusion of the kidneys.

·        Nephrotoxic acute tubular necrosis occurs as a result of direct damage to tubular cells by ingestion, injection or inhalation of a number of toxic agents. Toxic agents causing nephrotoxic acute tubular necrosis includes mercuric chloride, ethylene glycol, carbon tetrachloride etc.

 

Macroscopically, the kidneys are enlarged and swollen (edematous). On cut section, the cortex is pale, while the medulla is slightly darker than normal. The capsule can be easily removed.

64.

A 44 year  old  woman  complains  of general weakness, heart pain, significant increase   of   body   weight.   Objectively: moon  face, hirsutism,  AP  is 165/100 mm Hg, height  - 164 cm, weight - 103 kg; the fat  is mostly  accumulated on  her  neck, thoracic   girdle,  belly.  What is the main pathogenetic mechanism  of obesity?

Explanation

krushkrok No159 (2012)krushkrok No159a (2012)
65.

A  man  with  a  long-term  history  of bronchial asthma died from asphyxia. Histological  examination of his lungs revealed  that  the  lumens  of bronchioles and   minor   bronchi   contained  a  lot  of mucus  with some  eosinophils.  There  was also  sclerosis  of  interalveolar septa,  dilatation of alveoli lumens. What mechanism accounts for the development of hypersensitivity reaction?

Explanation

FullSizeRender (17)

Type I (Immediate, Anaphylaxis, Reagin): IgE (immunoglobulin E)-dependent activation of mast cells/basophils, usually accompanied by eosinophilia e.g. urticaria (hives), hay fever, asthma (wheezing), rhinitis and conjunctivitis (stuffy nose and itchy eyes; usually seasonal)

Type II (cytotoxic): antibody dependent reactions e.g. Goodpasture syndrome, Myasthenia gravis, Graves disease, ABO hemolytic disease of newborn etc.

Type III (immune-complex): deposition of antigen-antibody complexes e.g. systemic lupus erythromatous (SLE), Arthus reaction, serum sickness, poststreptococcal glomerulonephritis etc.

Type IV (cell mediated, delayed): antibody-independent T-cell mediated reactions e.g. positive mantoux reaction (tuberculin test), hashimoto’s thyroiditis or transplant rejection etc.

66.

A 36 year old female patient has a history of collagen disease. Urine analysis is likely to reveal an increased concentration of the following metabolite:

Explanation

Collagen, most abundant protein in human body; organizes and strengthens extracellular matrix. Collagen contains Gly-X-Y (X and Y are proline or lysine). Glycine(Gly) makes 1/3 of collagen. Oxyproline (hydroxyproline) is a major collagen amino acid which enables it to be regarded as a marker that reflects the catabolism of collagen. 

67. Bacterioscopy of  nasopharyngeal mucus  taken   from  a  2,5  year  old  child with nasopharyngitis revealed  gram- positive   diplococci   looking   like   coffee grains. What organs of the child are most likely to be affected if these microorganisms penetrate the blood?

Explanation

  Neisseria diplococcic are gram negative cocci that resemble paired kidney beans/coffee grains. The genus Neisseria contains two important human pathogens: Neisseria meningitides and Neisseria gonorrhoeae. Portal of entry is respiratory and genital tracts respectively. Meningococcal infection is an acute infectious process which has 3 main forms: nasopharyngitis; purulent meningitis and meningococcemia.

 

Meningitis is an acute or chronic inflammatory process chiefly affecting the pia and arachnoid mater (brain tunics) and cerebrospinal fluid. Meningococcal meningitis is characterized by the hyperemia of the pia mater, saturated with dull serous exudates during the first days of the disease.

68. Histologic  analysis  of uterus  mucous membrane  revealed   twisting  glands, serrated and spinned,  they were extended by stroma  growth with proliferation of its cells. Formulate a diagnosis:

Explanation

        Hyperplasia of endometrium is classified into 3 types:

·        Simple hyperplasia (cystic glandular hyperplasia)

·        Adenomatous hyperplasia (complex hyperplasia without atypia)

·        Atypical hyperplasia (complex hyperplasia with atypia)

 

Simple hyperplasia (cystic glandular hyperplasia) is characterized by the presence of large and cystically dilated varying-sized glands, which are lined by atrophic epithelium.

69.

Analysis of sputum taken from a patient with  suspected   pneumonia revealed rather elongated gram-positive diplococci with  somewhat   pointed  opposite   ends. What   microorganisms were  revealed   in the sputum?

Explanation

     There are two medically important genera of gram positive cocci: staphylococcus and streptococcus. Microscopically, staphylococci appear in grape-like clusters, whereas streptococci are in chains or pairs. Streptococcus pneumonia causes pneumonia, bacteremia, meningitis and infections of the upper respiratory tract. Pneumococci are gram positive lancet-shaped cocci arranged in pairs (diplococci) or short chains. The term “lancet-shaped” means that the diplococci are oval with somewhat pointed ends rather than being round).

 

Klebsiella and the Neisseria genus are gram negative.

70.

Serological diagnostics of infectious diseases  is based  upon  specific interaction  with  antigens. Specify  the  serological reaction that underlies adhesion of microorganisms  when  they  are  affected by  specific antibodies in presence of  an electrolyte:

Explanation

    Agglutination is commonly used as a method of identifying specific bacterial antigens and in turn the identity of such bacteria. Because the clumping reaction occurs quickly and is easy to produce, agglutination is an important technique in diagnosis. It is the clumping of cells such as bacteria or red blood cells in the presence of an antibody or complement. The antibody or other molecule binds multiple particles and joins them creating a large complex. The antibody (agglutinin) and antigen (agglutinogen) take part in the agglutination reaction. Their interaction takes place in definite quantitative proportions and in the presence of an electrolyte (0.85% NaCl solution). In mechanism and outer manifestation, the agglutination reaction is similar to the precipitin reaction. Both reactions are accompanied by the production of visible precipitates of antigen with the difference that in the agglutination reaction microbial bodies serve as the antigen, whereas in the precipitin reaction (precipitation reaction) the antigen is the product of the breakdown of microbial bodies, very minute particles of dissolved antigens requiring a large amount of antibodies for complete interaction.

71.

The immunoblot detected gp120 protein in the blood serum. This protein is typical for the following disease:

Explanation

      Human immunodeficiency virus (HIV): diploid genome (2 molecules of RNA). The 3 structural genes(i.e. proteins coded for by the genes) are:

·        env (gp 120 and gp 41): formed from cleavage of gp 160 to form envelope glycoproteins. gp 120 is for attachment to host CD4+ T cell. gp 41 is for fusion and entry.

·        Gag (p24): capsid protein

·        pol: reverse transcriptase, aspartate protease, integrase.

ELISA/Western blot (immunoblot) tests look for antibodies to the viral proteins listed above.

Reverse transcriptase synthesizes dsDNA (ds-double stranded) from genomic RNA (mRNA); dsDNA integrates into host genome. Virus binds CD4 as well as a coreceptor, either CCR5 on macrophages (early infection) or CXCR4 on I cells (late infection).

·        Homozygous CCR5 mutation – immunity

 

·        Heterozygous CCR5 mutation – slower course.

72.

HIV   has   gp41   and   gp120 on   its surface    interacts   with   target    cells   of an   organism.   Which   of   the   following human lymphocyte  antigens is gp120 complementary bound  with?

Explanation

      Human immunodeficiency virus (HIV): diploid genome (2 molecules of RNA). The 3 structural genes(i.e. proteins coded for by the genes) are:

·        env (gp 120 and gp 41): formed from cleavage of gp 160 to form envelope glycoproteins. gp 120 is for attachment to host CD4+ T cell. gp 41 is for fusion and entry.

·        Gag (p24): capsid protein

·        pol: reverse transcriptase, aspartate protease, integrase.

ELISA/Western blot (immunoblot) tests look for antibodies to the viral proteins listed above.

Reverse transcriptase synthesizes dsDNA (ds-double stranded) from genomic RNA (mRNA); dsDNA integrates into host genome. Virus binds CD4 as well as a coreceptor, either CCR5 on macrophages (early infection) or CXCR4 on I cells (late infection).

·        Homozygous CCR5 mutation – immunity

 

·        Heterozygous CCR5 mutation – slower course.

73.

A  65 year  old  patient with  chronic heart  failure  has been  taking  digitoxin  in self-administered dosages  for  a  long  time. She was admitted to the hospital for general health aggravation, arrhythmia, nausea,  reduced diuresis, insomnia.  What is the primary action to be taken?

Explanation

     Digitoxin is a long acting cardiac glycoside; used to treat chronic heart failure I-IIB stages, supraventricular tachyarrhythmia (fibrillation, Wolff-Parkinson-White syndrome). Since the patient has been taking this drug in  self-administered doses, there is a high probability of taking an overdosage which can result in toxic concentrations and at toxic concentrations, it causes ectopic ventricular beats that may result in ventricular tachycardia and fibrillation (arrhythmia). This arrhythmia is usually treated with lidocaine or phenytoin but in the absence of this, the first action to take is to stop taking the drug completely (to withhold digitoxin).

FullSizeRender (32)

74.

A 20 year old patient complains of morbid thirst and hyperdiuresis (up to 10 l daily). Glucose concentration in blood is normal but it is absent in urine. The patient has been diagnosed with diabetes insipidus.  What hormonal drug is the most appropriate for management of this disorder?

Explanation

      Diabetes insipidus (D.I.) is characterized by intense thirst and polyuria (hyperdiuresis) with inability to concentrate urine due to lack of ADH (central D.I.) or failure of response to circulating ADH (nephrogenic D.I.). in central D.I. antidiuretic hormone (ADH) is decreased; in nephrogenic D.I. ADH is normal. Therefore, the hormonal drug we can use to treat diabetes insipidus is to replace the low antidiuretic hormone (ADH) in central diabetes insipidus. So, we use ADH – a posterior pituitary hormone which increases water reabsorption in renal tubules and decrease dieresis. Its analogue desmopressin acetate can also be administered intranasally. Nephrogenic diabetes insipidus does not require hormonal replacement.

75.

A   patient has myocardial   infarction  with  thrombosis of  the  left  coronary artery. What pharmacological preparation group should be used to reestablish blood flow?

Explanation

The thrombosis in the left coronary artery has probably led to the development of myocardial infarction. Therefore, the best action to take to reestablish blood flow is to break up the thrombus. To achieve this, we need a fibrinolytic (thrombolytic) – which are used for dissolution of the blood thrombi by stimulating natural fibrinolytic system (activation of plasminogen to active plasmin which degrades fibrin).
76.

A nurse  accidentally  injected  a nearly double dose of insulin to a patient with diabetes  mellitus. The patient lapsed  into a hypoglycemic coma. What drug should be injected in order to help him out of coma?

Explanation

   Hypoglycemic coma is an acute condition progressing during rapid reduction of glucose level in blood and abrupt lowering of glucose utilization by brain. Overdosage of insulin is the most common cause; other causes are alcohol, overdose of oral hypoglycemic agents, insulinoma etc.

Treatment: for correction of hypoglycemia

20.0-60.0ml of 40% solution glucose intravenously (IV)

500ml of 5% solution glucose IV drip feed; or

Subcutaneous injection of glucagon 1.0ml

 

Subcutaneous injection of 1.0ml – 0.1% adrenaline (epinephrine)

77. A patient has a slowly healing fracture. What  medicine  can be used to accelerate formation of connective tissue matrix?

Explanation

      Methyluracil is a nonsteroidal anabolic, it can also act as a leucopoiesis stimulator, a non-specific drug which stimulates nucleic acid synthesis and proteins sufficient for leucopoiesis and other regeneration processes. Uses – radiation skin damages, fractures, slow recovering burns, agranulocytosis and leucopenia.

Cyclophosphan and Methotrexate are anticancer drugs

 

Prednisolone (glucocorticoids) and cyclosporine are immunosuppressants.

78.

From  the group  of children  who were eating  sweet  sappy  watermelon two kids developed the  signs  of  poisoning:  rapid weakness,  dizziness,  headache, vomiting, edema,   tachycardia,  cyanosis  of  mouth, ears, tips of the fingers cyanosis. High concentration of nitrates was detected. What is the leading mechanism of the pathogenesis of the  poisoning  in the  two children?

Explanation

    Oxidation of the heme component of hemoglobin to the ferric (Fe3+) state forms methemoglobin, which cannot bind O2. This oxidation may be caused by the action of certain drugs or substances containing nitrates, or endogenous products such as reactive oxygen intermediates. Furthermore a deficiency of NADH-met-hemoglobin reductase i.e. the enzyme responsible for the conversion of methemoglobin (Fe3+) to hemoglobin (Fe2+); leads to the accumulation of methemoglobin. The methemoglobinemias are characterized by “chocolate cyanosis” (a brownish-blue colouration of the skin and  membranes) and chocolate-coloured blood, as a result of the dark-coloured methemoglobin. Symptoms are related to tissue hypoxia and include anxiety, headache and dyspnea. Treatment is with methylene blue, which is oxidized as Fe3+  is reduced.

79. As a result of a trauma a patient has damaged  anterior  roots   of  spinal  cord. What structures have been affected?

Explanation

FullSizeRender (16) Anterior (ventral, motor) spinal cord root: arises from the anterolateral sulcus and contains a set of axons of motor neurons located within the anterior columns; the anterior roots number 31 pairs. The roots of the segments C8 through L2 also comprise the autonomic (sympathetic) fibers that arise from the sympathetic nuclei of the lateral grey columns.

80.

During a prophylactic medical examination   a  7-year-old   boy  was  diagnosed with  daltonism.   His parents are  healthy and have normal colour vision, but his grandfather on his mother’s side has the same abnormality. What is the type of the abnormality inheritance?

Explanation

image

It is sex linked, if only males or only females(X-linked) are affected.

Parents are healthy. Grandfather –Boy. A generation was left out (skipped).

Autosomal dominant: often due to defects in structural genes. Many generations, both male and female are affected. Found in every generation; no generation is left out (skipped). Parent – child in every generation.

Autosomal recessive is usually seen in some generations (other generations are skipped).

81.

The contents of vesicles that appeared on the  mucous  membrane  of  a  patient with variola was sent to a virological laboratory.  Which of the  listed  changes were revealed during the smear microscopy?

Explanation

    The pox virus family includes 3 viruses of medical importance: smallpox virus, vaccinia virus and molluscum contagiosum virus (MCV). Poxviruses are the largest and most complex viruses. The smallpox virus is also called variola virus. It is the agent of smallpox, the only disease that has been eradicated from the face of the earth. Eradication is due to the vaccine. Smallpox virus is transmitted via respiratory aerosol or by direct contact with virus either in the skin lesions or on fomites such as bedding. The virus replicates in the cytoplasm which form inclusion bodies (paschen) stainable in the cytoplasm of infected cells.

 

Negri bodies are inclusion bodies in Rabies; Guanieri bodies are inclusion bodies in vaccinia, variola (Note: vaccinia virus is nonpathogenic for humans); but paschen bodies is specific and pathognomonic for variola or smallpox.

82.

Vitamin  A together with specific cytoreceptors penetrates through the nuclear   membranes, induces  transcription  processes  that  stimulate growth  and differentiation of cells. This biological function  is realized  by the following form of vitamin A:

Explanation

Vitamin A and retinoids play an important role in the orderly differentiation of mucus-secreting epithelium. All-trans-retinoic acid, a potent acid derivative of vitamin A, has the highest affinity for retinoic acid receptors (RARs) compared with other retinoids. Activation of RARs by their ligands causes the release of corepressor and the obligatory formation of heterodimers with another retinoid receptor, known as the retinoic X receptor (RXR). The RAR/RXR heterodimers bind to retinoic acid response elements located in the regulatory regions of genes that encode receptors for growth factors, tumor suppressor genes and secreted proteins. Through these effects, they regulate cell growth and differentiation, cell cycle control and other biologic responses.
83.

In order to determine toxigenicity  of diphtheria bacilli  a  strip  of  filter  paper impregnated with antitoxic  diphtherial serum was put on the dense nutrient medium.  There   were  also  inoculated  a  microbial  culture  under  examination and  a strain  that  is known  to  be  toxigenic.  If the   microbial   culture   under   examination produces exotoxin, this  will result  in formation of:

Explanation

     The presence of the antitoxic diphtheria serum (antibody) and the exotoxin (antigen) cause an interaction between them leading to the production of antigen-antibody complexes producing precipitates (precipitin lines) on the strip of filter paper. We will have a precipitin ring if a tube were to be used instead of a filter paper.

84.

A   63 years   old   male   patient who had   been    suffering   from   chronic    diffuse obstructive disease, pulmonary emphysema, for 15 years died from cardiac insufficiency. Autopsy revealed nutmeg liver cirrhosis, cyanotic induration of kidneys   and   spleen,   ascites,   edemata of lower limbs. These changes of internal organs are typical for the  following  disease:

Explanation

 FullSizeRender (30)

     Acute heart failure refers to sudden and rapid onset of signs and symptoms of abnormal heart functions. Chronic heart failure is characterized by the symptoms that appear slowly over a period of time and become worst gradually.

The main symptoms of right-sided heart failure are fluid accumulation and swelling (edema) in the feet, ankles, legs, liver and abdomen. Left-sided heart failure leads to fluid accumulation in the lungs, which causes shortness of breath. At first, shortness of breath occurs only during exertion, but as heart failure progresses, it occurs with less and less exertion and eventually occurs even at rest. Moist and dry rales are heard over the lungs.

She has been suffering it for 15 years – chronic

Edema of lower limbs, ascites – Right ventricular insufficiency

85.

A man who is riding the carousel presents with increased heart rate, sweating, nausea.  This condition is caused primarily by the stimulation of the following receptors:

Explanation

     The receptor areas of the vestibular labyrinth are represented with the following structures:

·        Macula of Utricle

·        Macula of Saccule

·        Ampullary crests

 

The first two detect linear motion, while the ampullary crest detect angular motion. The vestibulocochlear nerve supply these receptors. Both maculae consist of the sensory hair cells covered with jelly-like susbstance. The substance contains the crystals of calcium carbonate called otoliths (vestibular otolith). The utricle and saccule detect linear movement, also contributing to balance. The ampullary crests reside within each membranous ampulla. They also comprise the sensory hair cells covered with the same jelly-like substance called the ampullary cupula. The ampullae is the sensory organ in the semicircular canal that sense angular (rotational) acceleration of the head, thereby regulating balance. NB: carousel is a revolving belt.

86.

A patient underwent esophagogastroduodenoscopy. Analysis of the  biopsy  material enabled doctors to diagnose him with helicobacteriosis. What property of the bacteria found  in this  patient had  to  be obligatory taken into account  during their cultivation?

Explanation

        Helicobacter pylori causes gastritis and peptic ulcers. Infection with H. pylori is a risk factor for gastric carcinoma and is linked to mucosal-associated lymphoid tissue (MALT) lymphoma. It is a gram negative, flexibacteria, oxidase positive, microaerophilic, motile and the only species in the Helicobacter genus to have multiple unipolar – sheathed flagella. Microaerophiles need O2 because they cannot ferment or respire anaerobically. However, they are poisoned by high concentrations of O2. They gather in the upper part of the test tube but not the very top. CO2 is essential for initial growth of H. pylori in liquid media (microaerophilic property). They produce urease, so it’s a diagnostic tool and not a necessary consideration for cultivation (culture medium).

87.

A 38 year old man died all of a sudden. Autopsy revealed   myocardial   infarction in the posterior wall of the  left  ventricle. What are the most likely alterations in  myocardiocyte structure  that   can  be revealed   microscopically   in  the  infarction focus?

Explanation

     In myocardial infarction there is necrosis. Nuclear changes in necrosis observed microscopically (light microscope) includes:

·        Karyopicnosis: at first nucleus shrinks and becomes dense.

·        Karyorrhexis: characterized by rupture of nuclear membrane and fragmentation of nucleus. Nucleus is decomposed into small granules.

·        Karyolysis: develops when nucleus dissolves

 

At electron microscope level, in addition to the above nuclear changes, disorganization and disintegration of the cytoplasmic organelles and severe damage of the plasma membrane are seen.

88.

During  cytoscopy  mucous  membrane of urinary  bladder normally  makes  folds except  for  a  single  triangular area  with smooth  mucosa. This triangle is located in the following part of urinary bladder:

Explanation

krushkrok No125 (2013)

The floor of the urinary bladder houses the trigone of urinary bladder. The trigone lacks the rugae as the submucosa is scarce here and mucosa adheres directly to the muscular layer. The vertices of trigone are related to the ureteric orifices and the internal urethral orifice. It is located at the fundus of bladder – lower wide and dense portion.

89. A child suffers from drug idiosyncrasy. What is the cause of such reaction?

Explanation

Hereditary enzymopathy is an inherited inefficiency or lack of enzymes participating in drug metabolism.

90.

Atria of an experimental animal were superdistended by blood that  resulted in decreased reabsorption of N a+ and water in renal tubules.  This can be explained by the influence of the following factor upon kidneys:

Explanation

      Antidiuretic hormone (vasopressin) is secreted in response to decrease blood volume and increase plasma osmolarity. It binds to receptors on principal cells of collecting ductules causing increase number of aquaporins and increase water reabsorption which leads to decreased diuresis.

 

Atrial natriuretic peptide is secreted in response to increase atrial pressure. It causes increase glomerular filtration rate (GFR) and increase sodium ion filtration with no compensatory sodium ion reabsorption and water in distal nephron which leads to increase diuresis.

91.

After a serious psychoemotional stress a 48 year old patient suddenly  developed acute heart   ache  irradiating  to  the  left arm. Nitroglycerine relieved  pain after 10 minutes. What is the leading pathogenetic mechanism of this process development?

Explanation

FullSizeRender (34)      Administered nitrates → ↑ Nitrites →↑ Nitric oxide (NO) → ↑cGMP → ↑dephosphorylation of myosin chains → vascular smooth muscle relaxation. Nitroglycerin is an organic nitrates. Nitrates inhibit coronary vasoconstriction or spasm, increasing perfusion of the myocardium and thus relieving vasospastic angina. Because of this action, nitrates are effective in treating effort-induced (stress) angina. Classic, effort-induced or stable angina are experienced in such conditions as physical activity, emotional excitement or any other cause of increased cardiac workload.
92.

A   42 year   old   patient complains of  pain  in  the   epigastral  area,   vomiting;  vomit   masses   have   the   colour   of \"coffee-grounds\", the patient has also melena.  Anamnesis records gastric ulcer. Blood formula:  erythrocytes - 2, 8 · 1012/l, leukocytes   - 8 · 109/l, Hb-  90 g/l. What complication is it?

Explanation

Principal complications of peptic ulcer are perforation, penetration, hemorrhage, stenosis, malignization (cancer). Gastric hemorrhage is a very important symptom. It can be manifested by vomiting blood (hematemesis) or by black tarry stools (melena). The vomitus looks like coffee grounds. If hemorrhage is profuse (damage to a large vessel) the vomitus contains much scarlet (unaltered) blood. Hematemesis occurs in peptic ulcer, cancer and polyps, in erosive gastritis etc.

93. A  56 year  old patient suffering  from cardiac  insufficiency  has  edema   of  feet and shins, edematous skin is pale and cold. What  is the  leading  mechanism  of edema pathogenesis?

Explanation

The main symptoms of right-sided heart failure are fluid accumulation and swelling (edema) of the feet, ankles, legs, liver and abdomen. Where the fluid accumulates depends on the amount of excess fluid and the effects of gravity. If a person is standing, fluid accumulates in the legs and feet; if a person is lying down, fluid usually accumulates in the lower back; if the amount of fluid is large, fluid also accumulates in the abdomen.

 

Most water leakage occurs in capillaries or postcapillary venules which have a semipermeable membrane wall that allows water to pass freely than proteins. In cardiac insufficiency, blood pools in the veins since the heart is not pumping effectively; this increases the hydrostatic pressure in the veins. And this increased hydrostatic pressure leads to increase filtration of water into the interstitium at the venous end of the capillary (post-capillary venule).

94.

A disaster fighter at a nuclear power plant developed  hemorrhagic syndrome on the  background of acute  radiation disease. What is the most important factor of syndrome pathogenesis?

Explanation

1.     Radiation disease, radiation therapy, chemotherapy or toxic chemicals can destroy megakryocytes. Megakaryocytes are responsible for the production of thrombocytes (platelets) which can lead to thrombocytopenia. Thrombocytopenia is an abnormally low level of platelets in blood, as a result of reduced platelet production in the bone marrow or excessive peripheral destruction of platelets. People with severe thrombocytopenia may have abnormal bleeding (hemorrhagic syndrome) almost anywhere in the body.

95. To prevent postoperative bleeding a 6-year old child was administered vicasol that  is a synthetic  analogue  of vitamin K . Name post-translational changes of blood coagulation factors  that  will be activated by vicasol:

Explanation

Prothrombin time (PT) evaluates the extrinsic coagulation system down to the formation of the fibrin clot. Factors that are evaluated include factor VII, X, V, II, and I. PT is increased when a factor level is 30% - 40% of normal. Vitamin K-dependent factors include factor II, VII, IX and X; protein C and S. vitamin K is activated in the liver by epoxide reductase. Activated vitamin K ɣ-carboxylates each of the vitamin K-dependent factors. Carboxylated factors are now able to bind calcium, which are essential for interaction between the coagulation factors and platelet membranes.

 

Warfarin inhibits the enzyme vitamin K epoxide reductase. Neonates lack enteric bacteria which produce vitamin K.

96.

While  under  barbituric anaesthesia a 65 year old male patient developed respiratory inhibition.  Anesthesiologist made him a 10 ml intravenous injection  of 0,5% bemegride solution.  The  patient’s  condition got better, the  pulmonary ventilation volume  increased.  What  phenomenon underlies the  interaction of  these  medications?

Explanation

1.     Barbiturates (Phenobarbital, thiopental) are hypnotics, sedative for anxiety, seizures, insomnia, induction of anesthesia and they induce normal sleep. They have narcotic type of action. Any degree of depression of the CNS is possible, depending on the dose.

Bemegride on the other hand is an analeptic. Analeptics are drugs which act upon the CNS general functions, but stimulates mainly medulla oblongata (respiratory, cardiovascular centers). They can be used in stimulation of respiratory in overdosage or poisoning with narcotic drugs (drugs for general anesthesia, barbiturates).

 

Both drugs act on CNS but with directly opposite (antagonistic) actions and with a direct action.

97. A  surgeon  has  to  find  the  common hepatic duct during the operative intervention on account of concrements in the  gall ducts. The  common  hepatic  duct is located  between the leaves of:

Explanation

krushkrok No95a (2012)   FullSizeRender (35)

Hepatoduodenal ligament running from the porta hepatica to the superior part of the duodenum. It consists of double layer of peritoneum and enfolds 3 structures:

·        Common hepatic artery

·        Portal vein

·        Common bile duct

Portal vein is localized posteriorly; hepatic artery – left anterior and bile duct – right anterior.

98. Blood    analysis   of   a   patient   with jaundice reveals conjugated bilirubinemia, increased concentration of bile acids. There  is no  stercobilinogen in urine. What type of jaundice  is it?

Explanation

 

Indirect; Hemolytic; Prehepatic

Mixed; Parenchymal; Hepatic

Direct; Obstructive; Mechanic; Posthepatic

Stercobilin (faeces)

        ↑↑↑

Decreases (pale faces)

Absent (clay coloured faeces)

Type of bilirubin in blood

Unconjugated

Conjugated and Unconjugated

Conjugated

Obturation (obstruction, to close) of bile duct – it can be:

* Intrahepatic – blockage of intrahepatic bile ducts

* Extrahepatic – blockage of common bile duct (ductus choledochus).

Findings:

* malabsorption: bile salts do not enter the Small Intestine; no emulsification of fat.

*light coloured stool: due to lack of urobilin (which leads to lack of stercobilin).

*Jaundice (posthepatic, mechanic, obstructive): increased conjugated Bilirubin.

* Steatorrhea

 

The findings are specific for obstruction of bile duct and bile acid deficiency.

99.

It is known that the gene responsible for development of blood groups according to  AB0  system  has  three  allele  variants. If a man has IV blood group, it can be explained by the following variability form:

Explanation

     Type IV blood group – AB

Combinative because both alleles of this gene are equally expressed in dominant forms and there is no masking of the characteristics (phenotype) of any of the allele.

100.

Power    inputs    of   a   man    were measured. In  what  state  was this  man  if his power  inputs  were  lower  than  basal metabolism?

Explanation

1.     Sleep is the natural periodic state of rest for mind and body with closed eyes characterized by partial or complete loss of consciousness. Sleep is unconsciousness from which the person can be aroused by sensory or other stimuli. It is to be distinguished from coma, which is unconsciousness from which the person cannot be aroused. During sleep, most of the body functions are reduced to basal level or even below the basal level. Therefore, power inputs can be lower than basal level during sleep. Unlike rest where it remains at the basal level or higher.

101. A  man  is  being  measured  power inputs  on an empty  stomach,  in the lying position,  under  conditions of physical and psychic rest at a comfortable temperature. Power inputs will reach the maximum at:

Explanation

1.     5-6pm

102.

When   measuring   power   inputs   of a man by the  method of indirect  calorimetry the following results were obtained: 1000 ml oxygen consumption and 800 ml carbon  dioxide liberation per minute.  The man under examination has the following respiratory coefficient:

Explanation

RQ = vCO2   /vO2 = 800 /1000 (ml) = 0.8

Minute O2 uptake – 1000ml

    Minute CO2 emission – 800ml

    Respiratory Quotient (RQ) = vCO2 /vO2 = 800/1000(ml) = 0.8

103.

A patient complains about  impaired evacuatory  function   of  stomach   (long- term retention of food in stomach). Examination revealed a tumour of initial part  of duodenum. Specify localization of the tumour:

Explanation

1.     The pylorus is a thickened part of the stomach which becomes continuous with the initial part of duodenum. The duodenum begins at the pyloric part of stomach and divided into 4 parts: superior part, descending part, inferior (horizontal) part and the ascending part. The superior part (pars superior) 5cm long, begins from the pylorus and runs horizontally and slightly backwards. On forming the superior duodenal flexure, the intestine passes into the descending part.

104.

As a result of damage to certain structures of brainstem an animal lost orientation reflexes.  What   structures were damaged?

Explanation

1.     Quadritubercular bodies/tectal plate is the dorsal portion of the midbrain comprises 4 colliculi – two superior and two inferior. The nuclei of colliculi are responsible for reflexes associated with sudden sound and visual stimuli (auditory and visual orientative reflexes); they also maintain consciousness. The nuclei give rise to the tectospinal tract. Superior colliculi – visual orientative reflex; Inferior colliculi – auditory orientative reflex.

105. Emotional stress causes activation of hormone-sensitive triglyceride lipase in the adipocytes.  What secondary mediator takes part in this process?

Explanation

The mobilization of stored fat requires the hydrolytic release of fatty acids and glycerol from their triacylglycerol (TAG) form. This process is initiated by hormone-sensitive lipase, which removes a fatty acid from carbon 1 and/or carbon 3 of the TAG. This enzyme is activated when phosphorylated by cyclic AMP (cAMP)-dependent protein kinase. cAMP is produced in the adipocyte when one of several hormones (such as epinephrine or glucagon – stress hormones) binds to receptors on the cell membrane and activates adenylyl cyclase.

106.

A  patient has  been  diagnosed  with alkaptonuria. Choose  an  enzyme  whose deficiency can be the reason for this pathology:

Explanation

image

Ochronosis (Alkaptonuria): congenital deficiency of homogentisate oxidase (homogentisic acid oxidase) in the degradative pathway of tyrosine to Fumarate → pigment-forming homogentisic acid accumulates (homogentisuria) in tissues. Autosomal recessive. Usually benign. Urine turns black on prolonged exposure to air. May have debilitating arthralgias (homogentisic acid toxic to cartilage).

107. A 10 year old child had the mantoux tuberculin  test   administered.  48  hours later  a  papule  up  to  8 mm  in  diameter appeared  on  the   site  of  the   injection. What type of hypersensitivity reaction developed after the tuberculin injection?

Explanation

Type IV (cell mediated, delayed): antibody-independent T-cell mediated reactions e.g. positive mantoux reaction (tuberculin test), hashimoto’s thyroiditis or transplant rejection etc.

The Mantoux skin test should be read between 48 and 72hrs after administration. The basis of reading  is the presence or absence of induration, which may be determined by inspection and by palpation. A record should also be made of formation of vesicles, bullae, lymphangitis, ulceration and necrosis at the test site. The formation of vesicles, bullae or necrosis at the test site indicates positive result. A negative mantoux result usually signifies that the individual has never been exposed to Mycobacterium tuberculosis i.e. absence of cell mediated immunity to tuberculin.

Mantoux test is a type IV Hypersensitivity reaction (HSR), which involves macrophages,T-lymphocytes and lymphokines(cytokines). Mononuclear cells (lymphocytes,monocytes,macrophages).

Remember,it is antibody independent (i.e does not involve antibodies).

B-lymphocytesàPlasma cellsàIg(Antibodies)------- none is involved in Type IV HSR.    
108.

The   patient  with  acute   miocardial infarction  was given intravenously different    solutions     during     8    hours with   medical    dropper   1500   ml   and oxygen   intranasally.  He   died   because of  pulmonary edema.   What  caused  the pulmonary edema?

Explanation

1.     Edema commonly results from increase in hydrostatic pressure or decreased oncotic pressure. With the infusion of different solutions intravenously amounting to 1500ml within 8hours, there is an increase in hydrostatc pressure, which results in the pulmonary edema. The myocardial infarction suffered by the patient aids in the pathogenesis of the pulmonary edema. Pulmonary edema is as a result of left ventricular insufficiency i.e. the myocardial infarction is in the left ventricular myocardium. ↑hydrostatic pressure = ↑volume

109.

In  course  of  an  operation surgeon removed a part  of a lung that  was ventilated  by a tertiary bronchus  accompanied by branches  of pulmonary artery  and other vessels. What part of a lung was removed?

Explanation

      The main (primary or first-order) bronchus branches in the hilum of lungs to form the lobar bronchi (secondary), which ventilate the respective lobe. The lobar bronchi in turn branch to form the tertiary (third-order) bronchi called the segmental bronchi. They ventilate the pulmonary areas called segments. Segments are the pyramidal shaped portions of lungs with apices facing the lung root and bases at the lung surface. Each segment excluding the segmental bronchus has a segmental branch of the pulmonary artery respective to segmental bronchi branches. The branches of pulmonary veins run between the segments. Ant segment is surgically removable without major damage to neighboring segments.

110. After  the  second  abortion a 23 year old   woman   has   been diagnosed with toxoplasmosis. Which drug should be used for toxoplasmosis treatment?

Explanation

     Toxoplasma gondi (a protozoa) causes toxoplasmosis, including congenital toxoplasmosis. Tissue cyst (pseudocysts) in undercooked meat or oocyst in cat faeces infects humans. Stages in human most associated with disease includes:

·        Rapidly multiplying trophozoites (tachyzoites) within various cell types.

·        Tachyzoites can pass placenta and infect fetus (congenital toxoplasmosis)

·        Slowly multiplying trophozoites (bradyzoites) in tissue cysts.

IMG_9914

Symptomatic and asymptomatic forms of congenital toxoplasmosis should be treated with a combination of sulfadiazine and pyrimethamine/Trimethoprim. Cotrimoxazole is a combination of sulfadiazine + Trimethoprim

Itraconazole – antifungal; Mebendazole – antihelminthic; Azidothymidine and acyclovir – antiviral.

111.

Autopsy of  a  17 year  old  girl  who died   from   pulmonary  failure   revealed a small area of caseous necrosis in the inferior  lobe of the right lung, and occurrences of caseous necrosis in the bronchopulmonary, bronchial  and bifurcational lymph nodes. What is the most probable postmortem diagnosis?

Explanation

         Types of tuberculosis:

o   Primary tuberculosis

o   Post primary tuberculosis: - secondary tuberculosis

   -Hematogenous tuberculosis

            Primary tuberculosis is the infection of an individual who has not been previously infected or immunized. Also called Ghon’s complex or childhood tuberculosis. The primary complex in lungs is usually located in the lower part of the right upper lobes or the upper part of the lower lobes in segments 3,8,9,10. (NB: 8,9,10 segments are in the inferior lobe of the lungs). Primary complex or Ghon’s complex consists of 3 components: pulmonary component; lymphatic vessel component and lymph node component.

 

            Secondary tuberculosis mainly affects the upper lobes of the lungs.

112.

After  4 months  of treatment for tuberculosis the patient began complaining of toes and fingers numbness, sensation of creeps. He was diagnosed with polyneuritis.  What  antituberculous drug might have caused these complications?

Explanation

The primary adverse effect of isoniazid is peripheral neuritis, manifesting as paresthesias of the hands and feet. This appears to be due to a relative pyridoxine deficiency.

Isoniazid is the hydrazide of isonicotinic acid and is a pyridine. Pyridine occurs in many important compounds including  azines and the Vitamins Niacin(B3) and Pyridoxine(B6). Therefore, isoniazid can be interfered with Vit.B3,B6 and even B1 metabolism by competing with them.

Isoniazid is a first line antituberculosis drug that inhibits the synthesis of mycolic acid. Vit B6 is needed for the transformation of tryptophan to Vit.B3.

Adverse reaction of Isoniazid: peripheral neuritis, optic neuritis, hepatitis and idiosyncratic hepatotoxicity. 

113.

Examination of a 55 year old woman revealed   under   the   skin of  submandibular area a movable slowly growing pasty formation with distinct borders 1,0x0,7 cm large.  Histological examination revealed lipocytes that form segments of different forms and sizes separated from each other by  thin  layers  of  connective tissue  with vessels. What is the most probable diagnosis?

Explanation

Lipoma is a benign tumor of fatty tissue. It is the most frequent soft tissue tumor, arising in subcutaneous regions at any site but most commonly on the back, shoulder and neck. It may develop in every site where there is fat tissue. Lipomas are encapsulated, usually small yellow node with distinct (clear) borders.

 

Liposarcoma and fibrosarcoma are malignant with no distinct borders. Angioma is tumor of vessels; Fibroma and Fibrosarcoma are tumors of connective tissue.

114. According to the phenotypic diagnosis  a female  patient has been  provisionally diagnosed with X-chromosome polysomia.  This  diagnosis  can  be  confi- rmed by a cytogenetic  method.  What karyotype will allow  to  confirm  the  diagnosis?

Explanation

Barr body is an inactive X-chromosome. A normal female has one barr body XX, a normal male has no barr body XY.

Trisomy XXXX (only one X is active in a female; therefore, 2 barr bodies)

KlinefelterXXY (one barr body)

Turner’s – XO (no barr body)

46 XX is a normal female sex chromosome. All other options are karyotypes for male patients.

115.

A  patient has  been  diagnosed  with acute  glomerulonephritis that  developed after   he  had  had   streptococcal  infection. It is most  likely that  the  affection  of basal  glomerular membrane is caused  by an allergic reaction of the following type:

Explanation

Acute poststreptococcal glomerulonephritis: most frequently seen in children. Occurs approximately 2 weeks after group A streptococcal infection of pharynx or skin. Resolves spontaneously. Type III hypersensitivity reaction (Immune complex). Presents with peripheral and periorbital edema, cola-coloured urine, hypertension. On immunofluorescent microscopy: granular appearance due to IgG, IgM and C3 deposition along glomerular basement membrane and mesangium. On electron microscopy: subepithelial immune complex humps. On light microscopy: glomeruli enlarged and hypercellular. 

116. An elderly patient has chronic constipations induced by large intestine hypotonia. What  drug  should  be administered?

Explanation

Constipations requires the use of laxatives that will stimulate the movement of food through gastrointestinal tract (GIT), produce soft formed stool and accelerate defecation. Bisacodyl is typically prescribed for relief of constipation and for the management of neurogenic bowel dysfunction as well as part of bowel preparation before medical examinations, such as for a colonoscopy. Bisacodyl works directly on the colon to produce bowel movement. It works by stimulating enteric nerves to cause colonic mass movements (contractions); removing the colon hypotonia.

117.

Examination  of  patients with  periodontitis revealed  the interdependence between  the   rate   of  affection   of  periodontal  tissues  and  the  amount of lysozymes in saliva and gingival liquid. These results can be obtained during studying  the  following  protection system of an organism:

Explanation

Innate or non-specific resistance consist of physical and chemical barriers such as skin, gastric acid, mucus or tears, saliva as well as cells and active mechanisms such as phagocytes, natural killer cells and the complement system. They defend an organism in non-specific form/manner, responding in the same fashion, regardless of what pathogen it is. Examples of physical barriers:

·        Skin: surface is made up of dead skin cells rich in keratin, which impedes microorganisms from entering the body – mechanical barrier

·        Gastric acid is a powerful defense against invading bacteria.

·        Saliva and tears contain antibacterial enzymes such as lysozyme, which destroy the cellular walls of bacteria.

 

·        Mucus is another defense; coating the mucus membranes and contains IgA antibodies (a component of specific/adaptive immune system).

118. An isolated  cell of human  heart automatically generates excitement impulses  with frequency of 60 times  per minute. This cell was taken from the following heart  structure:

Explanation

The sinoatrial (SA) node is a small strip of modified cardiac muscle, situated in the superior part of lateral wall of right atrium, just below the opening of superior vena cava. The fibers of this node do not have contractile elements. SA node is called the pacemaker because the rate of production of impulse (rhythmicity) is higher in SA node than in other parts. The rate is 60-100/min and higher during tachycardia when stimulated by sympathetic effects. Atrioventricular (AV) node is 40-60/min.

119.

A patient with epilepsy and depressive reaction has been  administered a drug that  reduced epilepsy  manifestations and improved  the patient’s psychic condition.

Explanation

Natrii valproas (sodium valproate) has antiseizure(antiepileptic) activity. It has proved to be the most effective is seizure states with a subcortical focus (e.g. absence seizures). It has also been effective in grand mal seizures but much less effective in partial seizures. It has anticonvulsant, antidepressant, psychotonic (improves psychic conditions) and cardioprotective effects.

 

Ethosuxemide and Phenobarbital, phenytoin are antiepileptic but no psychotonic effects. Amitriptyline is an antidepressant.

120. A patient diagnosed  with morphinism has been  admitted to the narcological department. A  doctor  noted  a  decrease in  pharmacological  activity   of  morphine.  Repetitive use  of  a  drug  may  result in tolerance to its effect, and this phenomenon is called:

Explanation

Morphine is the major analgesic drug contained in crude opium and is the prototype strong agonist. Morphine and several other opoids have high affinity for μ receptors, whereas other agents have varying affinities for δ and κ receptors. Long term chronic administration can result in physical and psychological dependence (addiction). Withdrawal produces a series of autonomic, motor and psychological responses that incapacitate the individual and cause serious (almost unbearable) symptoms. Repeated use produces tolerance to the respiratory depressant, analgesic, euphoric and sedative effects of morphine. Morphinism is excessive or uncontrolled usage of morphine.

121.

As a result of durative antibiotic therapy a 37 year  old patient developed intestinal  dysbacteriosis.  What   type   of drugs should be used in order  to normalize intestinal microflora?

Explanation

Eubiotics or probiotics aids the development and repopulation of intestinal microbial flora in the event of diarrhea and during treatment with antibiotics and sulfa drugs. A dietary supplement rich in highly concentrated live active cultures contains select high enzymatic function yeast cultures.

122.

A  man  got  poisoned  with mushrooms. They contain  muscarine  that stimulates muscarinic cholinoreceptors. What  symptom   is  typical  for  poisoning with inedible mushrooms?

Explanation

Muscarinic receptors belong to the class of G protein-coupled receptors. These receptors, in addition to binding Acetylcholine, it also recognizes muscarine – an alkaloid that is present in certain poisonous mushrooms. Muscarinic effects include miosis, spasm of accommodation, increasing of lacrimal, salivary, gastrointestinal, bronchial and sweat glands secretion, increase of smooth muscle tonus of internal organs, bradycardia, decreased arterial pressure.

 

All other options are sympathetic effects (acting through α or β-adrenergic receptors).

123.

A patient complains  of skin itch, especially between fingers, in the inguinal creases,  on  the  lower  abdomen. Examination   of  these   regions   revealed   there some small vesicles. Laboratory diagnostics allowed to establish  that  this condition had  been  caused  by  a  representative of Arthropoda. Specify  the  disease  caused by this arthropod:

Explanation

Scabies is a contagious skin disease caused by the itch mite – Sarcoptes scabiei. A person may become infected either by direct contact with a sick individual (including sexual contact) or by indirect one (when the bed clothes, underwear or bed is shared). Commonly the skin eruptions are localized on the interdigital folds of hands, on the lateral surfaces of fingers, flexible surfaces of wrist joint, on the skin of forearms, shoulders, chest, abdomen, buttocks and thighs. Itching is the first and essential symptom of scabies. The pruritic lesions result from a delayed hypersensitivity reaction to the faeces of the mite. The mite is located within the stratum corneum of the epidermis. At the sites of entry and exit of mites double morphologic elements of rash, more often microvesicles, papules, blisters or pustules, multiple scratches and excoriations and small sanguinolent crusts are formed.

124. An infant has pylorospasm, weakness, hypodynamia, convulsions as a result  of frequent vomiting.  What  kind of acid-base  disbalance  is it?

Explanation

Acidosis is the reduction in pH (increase in H+ concentration) below normal range. pH is less than 7.35; it is produced by:

·        Increase in partial pressure of CO2 in the body.

·        Decrease in HCO3- concentration.

Alkalosis is the increase in pH (decrease in H+ concentration) above normal range. pH is greater than 7.45; it is produced by:

·        Decrease in partial pressure of CO2 in the body.

·        Increase in HCO3- concentration.

Each of these two disorders has respiratory and non-respiratory forms. The non-respiratory form is divided into metabolic and excretory(non-gaseous).

·        Respiratory acidosis is the acidosis that is caused by alveolar hypoventilation e.g. airway obstruction due to bronchitis or lung diseases (pneumonia).

·        Respiratory alkalosis is caused by alveolar hyperventilation e.g. hypoxia in high altitude.

·        Non-respiratory:

-Metabolic acidosis is characterized by excess accumulation of organic acids such as lactic acid, ketoacids and uric acid formed by normal metabolism e.g. in Diabetes mellitus or extreme/prolonged exercise.

-Excretory/Non-gaseous acidosis may develop in impaired renal H+ excretion related to increased loss of bicarbonate in urine; diarrhea causes acidosis by the loss of bicarbonate with faeces.

-Excretory/Non-gaseous alkalosis: vomiting (loss of gastric acid), increased metabolism of lactate and citrate (turns into bicarbonate and water), long-term use of thiazides and loop diuretics.

 

      It is excretory/non-gaseous alkalosis because of the frequent vomiting.

125.

During  influenza  epidemic  40%  of pupils  who  didnt go  in  for  sports  were affected  by  the  disease,  and  among  the pupils who regularly did physical exercises this index  was only 20%. What  adaptative mechanisms  determined such a low sicknesrate  of pupils  participating in the sports?

Explanation

126.

A  39 year  old  man  who  had  been operated for the stomach ulcer died 7 days after  the  surgery.  Autopsy revealed  that peritoneal leaves were dull, plephoric, covered  with  massive  yellow-greenish films,  the  peritoneal cavity  contained for about  300 ml of thick yellow-greenish liquid.  What  pathologic process  was revealed  in the peritoneal cavity?

Explanation

Fibrinopurulent peritonitis: Supurative or purulent inflammation is characterized by the production of large amounts of pus. The cellular inflammatory response in peritoneal cavity is composed primarily of dense collections of neutrophils and fibrinopurulent debris that coat the visceral and abdominal wall. Serous peritonitis – thin fluid (not pus);

Hemorrhagic peritonitis  - hemorrhage; Tuberculous peritonitis – the patient must have TB, before there can be an extrapulmonary TB.

127.

Preventive  examination  of  a  patient  revealed  an enlarged lymph  node  of metastatic origin on the medial wall of the left axillary crease. Specify the most likely localization  of the primary tumour:

Explanation

Metastases of breast cancer is either local or distant. Local metastases is usually to the lymphatic nodes of breast base, axilla, subclavicular, parasternal nodes. Distant metastases are hematogenic ones, 40-50% to the bones, lungs, and liver.

Cancer of any of the following will metastasize locally through:

            Submandibular salivary gland – submandibular nodes

            Thyroid gland – deep anterior cervical nodes

            Stomach – celiac nodes

 

            Lungs – visceral thoracic lymph nodes

128.

A  60  year  old  patient  was  found to  have  a  dysfunction   of  main  digestive enzyme  of saliva. This  causes  the  disturbance of primary hydrolysis of:

Explanation

Saliva contains two major types of protein secretion:

·        A serous secretion that contains ptyalin (α-amylase or salivary amylase), an enzyme for digesting starches.

·        Mucus secretion that contains mucin for lubricating and for surface protective purposes.

 

Salivary amylase is a carbohydrate-digesting (amylolytic) enzyme. It acts on cooked or boiled starch and converts it into dextrin and maltose. Though starch digestion starts in the mouth, major part of it occurs in stomach because food stays only for a short time in the mouth. Optimum pH necessary for the activation of salivary amylase is 6. Salivary amylase cannot act on cellulose.

129.

A  patient suffers  from  stenocardia and takes isosorbide  mononitrate. He was prescribed a complementary drug with disaggregating  effect. What drug is it?

Explanation

Acetylsalicyclic acid (Aspirin) is an NSAID. Cycloxygenase, the enzyme which converts arachidonic acid into the endoperoxide precursors of prostaglandin, has at least two different  isoforms: COX-1 & COX-2. COX-1 is primarily expressed in non-inflammatory cells whereas COX-2 is expressed in activated lymphocytes, polymorphonuclear cells and other inflammatory cells. Acetylsalicyclic acid and the older non-selective NSAIDs inhibit both cyclooxygenase isoforms & thereby decrease prostaglandin & thromboxane synthesis (disaggregating effect) throughout the body.

130.

A 49 year old woman  spent  a lot of time standing.  As a result of it she got leg edema.  What  is the  most  likely cause  of the edema?

Explanation

The main symptoms of right-sided heart failure are fluid accumulation and swelling (edema) of the feet, ankles, legs, liver and abdomen. Where the fluid accumulates depends on the amount of excess fluid and the effects of gravity. If a person is standing, fluid accumulates in the legs and feet; if a person is lying down, fluid usually accumulates in the lower back; if the amount of fluid is large, fluid also accumulates in the abdomen.

 

Most water leakage occurs in capillaries or postcapillary venules which have a semipermeable membrane wall that allows water to pass freely than proteins. In cardiac insufficiency, blood pools in the veins since the heart is not pumping effectively; this increases the hydrostatic pressure in the veins. And this increased hydrostatic pressure leads to increase filtration of water into the interstitium at the venous end of the capillary (post-capillary venule).

131.

A patient presented to a hospital  with complaints about  quick fatigability and significant  muscle  weakness.  Examination revealed  an autoimmune disease  that causes  functional   disorder  of  receptors in the  neuromuscular synapses.  This  will result  in  the  disturbed activity  of  the following mediator:

Explanation

 Myasthenia gravis is an autoimmune disease of neuromuscular junction caused by antibodies to cholinergic receptors. It is characterized by grave weakness of the muscle due to the inability of neuromuscular junction to transmit impulses from nerve to the muscle. It is caused due to the development of auto-antibodies (IgG auto-antibodies) against the receptors of acetylcholine. That is, the body develops antibodies against its own acetylcholine receptors. These antibodies prevent binding of acetylcholine with its receptors or destroy the receptors. So, though the acetylcholine release is normal, it cannot execute its action.

132.

Blood   test   of  a  patient  suffering from  atrophic gastritis  gave  the  following results: RBCs - 2, 0 · 1012/l, Hb- 87 g/l, colour  index  - 1,3, WBCs  - 4, 0 · 109/l, thrombocytes - 180 · 109/l. Anaemia  might have been caused by the following substance  deficiency:

Explanation

Pernicious anemia or addison’s anemia is the anemia due to deficiency of vitamin B12. It is due to atrophy of the gastric mucosa (atrophic gastritis) because of autoimmune destruction of parietal cells. The gastric atrophy results in decreased production of intrinsic factor and poor absorption of vitamin B12, which is the maturation factor for red blood cells (RBC). RBCs are larger and immature with almost normal or slightly low hemoglobin level. Synthesis of hemoglobin is almost normal in this type of anemia. So, cells are macrocytic and normochromic/hypochromic.

133.

A 45 year old man consulted  a doctor about a plaque-like formation on his neck. Histological   examination  of  a  skin  bioptate  revealed   clusters   of  round   and oval tumour cells with a narrow  border of basophilic  cytoplasm  resembling of cells of basal epidermal layer. What  tumour is it?

Explanation

Skin cancer (epithelial tumors) is divided into 4 groups:

·        Intraepidermal cancer (cancer in situ): Bowen’s disease, erythroplasia of Queyrat.

·        Basal cell carcinoma (BCC, basalioma)

·        Epidermoid cancer

·        Skin appendage cell carcinoma: Paget’s disease

 

Basal cell carcinoma (basalioma) is malignant; the tumor is characterized by slow growth (sometimes for years), locally destructive invasive growth without metastases and frequent recurrences. The tumor is of polymorphous structure derived from the epidermis and skin appendages. The most prevalent location is the face, neck and scalp. Gradually increasing in size, they can form a large plaque or node with ulcerative depression in the center and elevated borders. It has the appearance of a flat well-defined reddish, round or oval-shaped spot or plaque.

134.

A 71 year old man had been presenting with diarrhea for 10 days.  The feces had admixtures of blood and  mucus.  He was delivered to a hospital in grave condition and died 2 days later. Autopsy of the body revealed   the following:  diphtheritic colitis with multiple irregularly-shaped ulcers of different depth  in both  sigmoid colon and rectus.  Bacteriological analysis revealed Shigella. What  was the main disease?

Explanation

Dysentery refers to bloody diarrhea with mucus. It refers to diarrhea with abdominal cramping and tenesmus in which loose stools contain blood, pus and mucus. Bacillary dysentery is caused by shigella dysenteriae, Shigella flexneri, Shigella boydii and Shigella sonnei. Morphology: it has 4 stages namely: catarrhal colitis; fibrinous colitis; ulcer formation (ulcerative colitis); healing of the wound. Complications of dysentery includes perforation, intraintestinal hemorrhage and eventually death may result from intestinal or extraintestinal complications.

135.

In spite of treatment with cardiotonics and thiazide diuretic  a patient suffering from chronic  cardiac  failure  still presents with edemata and  faces a risk of ascites. What  medication should  be administered in order  to increase  the diuretic  effect of the above mentioned drugs?

Explanation

IMG_9896

K+ sparing diuretics:

  - aldosterone antagonist: spironolactone, eplerenone

  - inhibit Na+ reabsorption: triamterene

The answer is Spironolactone because after the administration of cardiotonics and a thiazide diuretic, if the edema still persists, it is probably due to an increased action of aldosterone on the principal cells in collecting tubule of the kidney. In this case, an aldosterone antagonist will enhance the diuretic effect to prevent the occurrence of ascites. Blocking aldosterone effect prevent further reabsorption of Na+ and H2O which will definitely enhance the diuretic effect. Aldosterone acts on mineralocorticoid receptor → mRNA → protein synthesis (synthesis of Na+ channels). Spironolactone prevents/blocks the synthesis of Na+ channels.

     Carbonic anhydrase inhibitor: acetazolamide; Thiazide diuretics: hydrochlorthiazide; Loop diuretics: furosemide

136.

Autopsy  of  a  75  year  old  patient who  had   been   suffering   from   disseminated  atherosclerosis and died under chronic cardiac failure revealed  constriction and deformation of coronary  arteries, tuberous intima  whose  section  appeared to be white and petrosal. Specify the stage of atherosclerosis morphogenesis:

Explanation

Stages of atherosclerosis:                                                                                    àPre-lipid stage: mucoid swelling  of intima, destruction of endothelium, elastic & collagen fibres of intima’s basal membrane.

àStage of fatty stripes (lipidosis):  Fatty stripes appear on intima. Macrophages that have accumulated lipid in their cytoplasm appear as csantomic or foam cells.                                                                                      

 àStage of liposclerosis: Lipid accumulates  not only in macrophages but also in smooth muscle cells. In areas of lipidosis (lipid core), a young CT grows & forms a fibrous cap/plaque.                                                       

 àStage of atheromatosis: Necrosis of central part of fibrous cap, with formation of atheromas. Atheromas consist of  amorphous lipid-rich material & are soft.                                                                                             

 àStage of ulceration: Break of fibrous cap cover & ulceration with small hemorrhage into plaque.                                                                                     

àStage of atherocalcinosis: Deposition of calcium in ulcerative plaque. Calcification of vessels leads to hardening of arteries, which appears white & petrosal.

Only stage of liposclerosis and atherocalcinosis have been mentioned in Krok so far.

137. Examination of a bronchial  tissue sample revealed atrophy  of mucous membrane, cystic degeneration of glands, focal  metaplastic changes  of  lining  prismatic epithelial cells into multilayer squamous cells; increase in goblet cell number;  in some  parts  of bronchial wall and  especially  in the  mucous  membrane there  was marked cellular inflammatory infiltration and growth of granulation tissue bulging into the bronchial lumen in form of  a polyp. What is the most likely diagnosis?

Explanation

The hallmark and earliest feature of chronic bronchitis is hypersecretion of mucus in large airways and is associated with hypertrophy of the submucosal glands in the trachea and bronchi. As chronic bronchitis persists, there is also a marked increase in goblet cells of small airways leading to excessive mucus production that contributes to airway obstruction. Histological features of the small airways:

·        Goblet cell metaplasia with mucus plugging of the lumen.

·        Inflammatory infiltration

·        Fibrosis of bronchiolar wall (granulation tissue)

 

·        Atypical metaplasia and dysplasia of the respiratory epithelium, providing a possible soil for cancerous transformation.

138.

Acute    renal    impairment   caused death  of a bleeding patient. Autopsy revealed  enlarged kidneys  with  a  broad pale pink cortical layer expressively demarcated from  dark  red  renal  pyramids.   Macroscopic    examination  revealed lack of epithelial nuclei of convoluted tubules,  tubulorrhexis,  phlebostasis. The cell nuclei of choroid  glomus and straight tubules  were  present. What  pathology is it?

Explanation

     Acute tubular necrosis/Necrotic nephrosis/Necronephrosis involves the death of tubular epithelial cells that form the renal tubules of the kidneys. Most common cause of acute renal failure. It can be ischemic or nephrotoxic.

·        Ischemic acute tubular necrosis occurs due to hypoperfusion of the kidneys.

·        Nephrotoxic acute tubular necrosis occurs as a result of direct damage to tubular cells by ingestion, injection or inhalation of a number of toxic agents. Toxic agents causing nephrotoxic acute tubular necrosis includes mercuric chloride, ethylene glycol, carbon tetrachloride etc.

 

Macroscopically, the kidneys are enlarged and swollen (edematous). On cut section, the cortex is pale, while the medulla is slightly darker than normal. The capsule can be easily removed.

139.

Osteolaterism is charcterized by a decrease in  collagen  strength   caused  by much  less  intensive  formation  of  cross- links in collagen fibrils. This phenomenon is caused by the low activity of the following enzyme:

Explanation

Osteolaterism results in conditions of deficiency or non-functional lysyl oxidase. Lysyl oxidase is involved in the cross-linking of elastin and collagen resulting in disorganized connective tissue formation. Lysyl oxidase is an extracellular copper enzyme that catalyzes formation of aldehydes from lysine residues in collagen and elastin precursors. This results in cross-linking collagen and elastin, which is essential for stabilization of collagen fibrils and for the integrity and elasticity of mature elastin.

140.

A    histological    specimen    of   kidney shows a structure consisting of a glomerulus  of fenestrated capillaries  and a  bilayer  epithelial capsule.  Specify  this structure:

Explanation

krushkrok No140 (2012)

The nephron consists of the renal corpuscle and a tubule system. The renal corpuscle represents the beginning of the nephron. It consists of the glomerular capillary tuft which possesses numerous fenestrations and the surrounding visceral and parietal epithelial layers (bilayer) of Bowman’s capsule. Bowman’s capsule is the initial portion of the nephron where blood flowing through the glomerular capillaries undergoes filtration to produce the glomerular ultrafiltrate.

141.

A   66   year    old   female    patient got   intravenous  injection    of   magnesium sulfate solution for the purpose of elimination of hypertensive crisis. But arterial  pressure  didn’t   go   down   and after  repeated introduction of  the  same preparation there  appeared sluggishness, slow response, inhibition  of consciousness and   respiration.   What    preparation   is  antagonist of magnesium  sulfate  and  can eliminate symptoms of its overdose?

Explanation

Magnesium sulfate reduces striated muscle contractions and blocks peripheral neuromuscular transmission by reducing acetylcholine release at the neuromuscular junction. Magnesium is a unique calcium antagonist as it can act on most types of calcium channels in vascular smooth muscle and as such would be expected to decrease intracellular calcium. One major effect of decreased intracellular calcium would be inactivation of calmodulin-dependent myosin light chain kinase activity and decreased contraction, causing arterial relaxation that may subsequently lower peripheral and cerebral vascular resistance, relieve vasospasm and decrease arterial blood pressure. Therefore, it is recommended that intravenous calcium gluconate or calcium chloride should be available as an antagonist of magnesium to increase intracellular calcium level.

142.

As a result  of a road  accident  a 26- year-old   man  is  in  the  torpid   phase  of shock.  Blood   count:   leukocytes   -  3, .109/l. What  is the  leading  mechanism  of leukopenia development?

Explanation

Shock is a form of stress which is accompanied by the release of stress hormones (catecholamines, cortisol). Stress-induced changes in blood leukocyte distribution may represent an adaptive response. This represents a redistribution of leukocytes from the blood to other organs such as the skin, draining sentinel lymph nodes and other compartments. Such leukocyte redistribution may enhance immune function in compartments to which immune cells traffic during stress.

143.

A  5-month-old  boy  was  hospitalized  for  tonic  convulsions.  He  has  a life- time  history  of this disease.  Examination revealed  coarse  hair,  thinned and  fragile nails,  pale  and  dry  skin.  In  blood:  calcium - 1,5 millimole/l, phosphor - 1,9 millimole/l. These changes are associated with:

Explanation

Parathyroid hormone: secreted by chief cells of parathyroid gland. Effects include:

↑bone resorption of Ca2+ and PO43- → ↑their plasma levels

↑kidney reabsorption of Ca2+ in distal convoluted tubule → ↑ Ca2+ plasma level

↓reabsorption of PO43- in proximal convoluted tubule → ↓ PO43- plasma levels

↑Calcitriol (vit D3) production by stimulating kidney 1α-hydroxylase in proximal convoluted tubule. It increases Ca2+ and PO43- absorption in the intestine.

In general, parathyroid hormone ↑ Ca2+ plasma level but ↓ PO43- plasma levels. Abnormal synthesis (↑synthesis) of parathyroid hormone can lead to hypercalcemia and hypophosphatemia.

Calcitonin is secreted by parafollicular cells (C cells of the thyroid gland). It ↓bone resorption of Ca2+. It opposes actions of parathyroid hormone. But its not important in normal Ca2+ homeostasis. Calcitriol ↑ circulating Ca2+ ions as a means of enhancing intestinal absorption of calcium (NB: Calcitriol production is dependent on parathyroid hormone).

Therefore hypoparathyroidism results in hypocalcemia and hyperphosphatemia.

144.

As a result of a trauma a patient has developed traumatic shock that led to the following disorders:  AP is 140/90 mm Hg, Ps is 120 bpm. The patient is fussy, talkative, pale. Such state relates to the following shock phase:

Explanation

Pirogov’s stages of shock

* Erectile phase: is characterized by strong motor agitation, sweating, tremor of skeletal muscles, staggering gait, frequent urination, transient increase in blood pressure, heart rate and breath rate increases, body temperature also. Painful impulses reach CNS.

 

* Torpid phase: decompensation in CNS leads to deep oppression. Patient is motionless, does not answer questions or answers very silently and with long time of delay, reflexes are lowered or absent.

145.

Cytogenetic examination of  a  patient  with dysfunction  of the  reproductive system revealed  normal  karyotype 46,ХУ in some cells, but most cells have Klinefelter’s syndrome  karyotype - 47,ХХУ. Such phenomenon of cell inhomogeneity is called:

Explanation

FullSizeRender (1)

krushkrok No70 (2007)

Barr body is an inactive X-chromosome. So a boy (XY) with an inactive X-chromosome must have an additional X-chromosome – XXY (Klinefelter’s syndrome). Causes :

* nondisjunction (maternal and paternal nondisjunction in meiosis I)

* Mosaicism: with the karyotype being 46, XY/47, XXY

Manifestations: gynecomastia, female pattern of pubic hair distribution, no facial hair, high voice.

146.

An oncological  patient had been administered methotrexate. With time target cells of the tumour lost sensitivity to this drug. At the same time the change in gene expression of the  following  enzyme is observed:

Explanation

image

IMG_9914

Methotrexate is structurally related to folic acid and acts as an antagonist of that vitamin by inhibiting dihydrofolate reductase (in humans), which is the enzyme that converts folic acid to its active coenzyme form, tetrahydrofolic acid. Trimethoprim and pyrimethamine inhibit the same enzyme but in bacteria and protozoa respectively. This decreases dTMP, needed for DNA synthesis.

147.

A 64 year old woman has impairment of twilight vision (hemeralopy). What  vitamin should be recommended in the first place?

Explanation

Rhodopsin or visual purple is the photosensitive pigment of rod cells. It is made up of a protein called opsin and a chromophore. Opsin present in rhodopsin is known as scotopsin. Chromophore is a chemical substance that develops colour in the cell. Chromophore present in the rod cells is called retinal. Retinal is the aldehyde of vitamin A or retinol. Vitamin A is the name given to a group of related compounds that include retinol (vitamin A alcohol); retinal (vitamin A aldehyde) and retinoic acid (vitamin A acid). Rod cells are responsible for dim light vision or night vision or scotopic vision.

Impairment of twilight vision is a defective function of rod cells. Therefore, vitamin A is recommended as the first choice.

148.

A patient underwent appendectomy. In the  postoperative period  he has been taking an antibiotic.  The patient complains  about  hearing  impairment and  vestibular disorders.  What group of antibiotics has such by-effects?

Explanation

Aminoglycoside binds the 30S ribosomal subunit and interferes with assembly of the functional ribosomal apparatus and/or can cause the 30S subunit of the completed ribosome to misread the genetic code. The elderly are particularly susceptible to nephrotoxicity and ototoxicity as an adverse effect of aminoglycosides. Ototoxicity (vestibular and cochlear) is directly related to high peak plasma levels and the duration of treatment. The antibiotic accumulates in the endolymph and perilymph of the inner ear and toxicity correlates with the number of destroyed hair cells in the organ of corti. Deafness may be irreversible and has been known to affect fetuses in utero.

149.

While  playing  a  child  got  a  punch in  the   presternum  region.   As  a  result of  this  trauma an  organ  located  behind the presternum was damaged.  Name  this organ:

Explanation

The thymus resides in the anterior mediastinum posterior to the manubrium of sternum reaching the IV rib. Mediastinal pleurae neighbor the thymus laterally, the pericardium, arch of aorta with associated branches and inferior vena cava posteriorly. Topographically, the thymus is closer to the sternum than the heart. So it will be affected first by a blow to the sternum because it lies outside of the pericardial cavity (anterior to the fibrous pericardium).

150. In  an  embryo  the  process  of dorsal mesoderm segmentation and somite formation  is  disturbed.  What   part   of skin will probably  have developmental abnormalities?

Explanation

The embryonic period or period of organogenesis occurs from the third – eighth weeks of development and is the time when each of the 3 germ layers: ectoderm, mesoderm and endoderm, give rise to a number of specific tissues and organs.

Mesoderm: paraxial, intermediate and lateral mesoderm. By the beginning of the 3rd week, paraxial mesoderm begins to be organized into segments. These segments, known as somitomeres, which further organize into somites (42-44 pairs). Somites: sclerotome (tendon, cartilage and bone – vertebrae and rib cage); Myotome (muscles); Dermatome (dermis). From their initial location within the somite, the sclerotome cells migrate medially towards the notochord.
151.

A patient consulted  a dentist  about itching and burning in the oral cavity; high temperature. The  patient was diagnosed with trichomonal gingivostomatitis. What drug should be chosen for his treatment?

Explanation

Metronidazole forms toxic free radical metabolites in the bacterial cell that damage DNA. It is bactericidal, and an antiprotozoal. It is used to treat Giardia, Entamoeba, Trichomonas, Gardnerella vaginalis, Anaerobes (bacteroides, Clostridium difficile). It can be used with a proton pump inhibitor and clarithromycin for “triple therapy” against Helicobacter pylori.

152.

An  electron microphotography of a fragment of  proper gastric  gland  shows a big irregular round-shaped cell. There are  a lot of intracellular tubules  and  mitochondria in the  cytoplasm.  Specify this cell:

Explanation

Parietal (oxyntic) cells are found in the neck of the fundic glands, among the mucus neck cells and in the deeper part of the gland. They secrete HCl and intrinsic factor. When examined with the transmission electron microscope they are seen to have an extensive intracellular canalicular system that communicates with the lumen of the gland. Also an elaborate tubulo-vesicular membrane system is present in the cytoplasm adjacent to the canaliculi. Numerous mitochondria with complex cristae and many matrix granules supply the high levels of energy necessary for acid secretion.

153.

A patient suffering from stenocardia was  taking  nitroglycerine which  caused restoration of blood  supply  of myocardium and relieved  pain in the cardiac  area. What intracellular mechanism  provides restoration of  energy  supply  of  insulted cells?

Explanation

Energy supply to any cell is provided by ATP. Nitroglycerine relieves stenocardia by causing vasodilation thereby increasing blood supply locally to the myocardium. This increase in blood supply, increases the supply of substrates needed to make ATP. Therefore, a dose of Nitroglycerine → ↑blood supply → ↑substrates for ATP synthesis → ↑intensification of ATP synthesis.

154.

A 50 year old patient has been  taking treatment thrice  for the last 6 months because  of fractures caused  by domestic accidents.   Microscopical   examination of bony tissue revealed foci of lacunar resolution, giant-cell granulomas in the tumour-like formations, cysts. Bony tissue was substituted by fibrous  connective tissue. Examination revealed  also adenoma of parathyroid gland  and  hypercalcemia. What is the most probable diagnosis?

Explanation

The main reasons for hypercalcemia are primary hyperparathyroidism (which can result from adenoma of parathyroid gland), hypervitaminosis D (calcium absorption in the intestine increases). Hypercalcemia often accompanies difficult fractures and this numerous fractures violate the equilibrium between bone construction processes, which sharply decreases and the resorption, which continues with previous speed. Reklinghausen described development of osteodystrophy as the result of bone decalcinosis at the continuous hyperparathyroidism (parathyroid adenoma). NB: parathyroid hormone → ↑bone resorption. This disease is characterized by gradual dilution and softening of bones. Metabolic osteoclasts activity increases due to the action of parathormone. Loss of bone matrix occurs and the bone becomes soft. Resorption allotments are replaced with fibrous connective tissue and zones of osteoid substance are formed.

155.

A  patient has  been  diagnosed   with  a  compression fracture  of  a  lumbar vertebra.  As  a  result   he  has  a  considerable  increase    in   curvature   of   the lumbar  lordosis.  Which ligament  damage can induce such changes in the spine curvature?

Explanation

Lordosis is an exaggerated anterior curvature of the spine, most often lumbar. Lordosis is present in the cervical and lumbar regions (cervical and lumbar lordoses). The intervertebral synchondroses and symphyses are reinforced by the longitudinal ligaments which run along the entire spine.

·        *Anterior longitudinal ligament is a band which extends from the atlas to the pelvic surface of the hip bone along the anterior surfaces of the vertebral bodies.

·     *Posterior longitudinal ligament runs along the posterior surface of the vertebral bodies (in the vertebral canal).

 

Therefore, a damage to the anterior longitudinal ligament can induce lordosis.

156.

When  water  affects  mucous membrane of the inferior  nasal meatuses, this causes \\\"diver reflex\\\" that provokes:

Explanation

When water or any irritating noxious vapors or gases affects the mucus membrane of the nose, an involuntary temporary arrest of breathing (apnea) develops. This is a protective reflex to prevent the subject from inhaling harmful substances (reflectory apnea – reflex apnea).

157.

Life   cycle   of   a   cell   includes   a process   of  DNA   autoreduplication.  As a result of this process monochromatid chromosomes become  bichromatid. This phenomenon is observed within the following period  of the cell cycle:

Explanation

Somatic cell division is a cyclic process divided into two phases: mitosis (M phase) and interphase. Three other phases, Gap 1 (G1); synthesis phase (S) and Gap 2 (G2) further subdivide interphase. M phase is followed by G1.

M phase:* karyokinesis – division of the nucleus into two daughter nuclei.

                  *Cytokinesis – division of the cell into two daughter cells.

 

G1: a period in which no DNA synthesis occurs. Period of cell growth S or DNA synthesis phase follows G1 phase. The DNA of the cell is doubled. The S phase is followed by a period in which no DNA synthesis occurs, a second gap or G2 phase. A cell that leaves the cycle in G1 phase to begin “terminal” differentiation enters the G0 phase, (“O” stands for “outside” the cycle).

158.

Examination  of  a  child  who frequently suffers from infectious diseases revealed  that  IgG concentration in blood serum was 10 times less than normal,  IgA and  IgM  concentration was  also  significantly reduced. Analysis showed also lack of B-lymphocytes and plasmocytes.  What disease are these symptoms typical for?

Explanation

X-linked (Bruton) agammaglobulinemia: defect in Bruton’s tyrosine kinase (BTK), a tyrosine kinase gene → no B cell maturation. X-linked recessive (↑in boys). Findings: absent B cells in peripheral blood, ↓immunoglobulin of all classes. Absent/scanty lymph nodes and tonsils.

159.

Examination of a 42 year old patient revealed   a  tumour of  adenohypophysis. Objectively:   the  patient’s  weight  is  117 kg, he has moon-like hyperemic  face, red- blue striae  of skin distension  on his belly.  Osteoporosis and muscle dystrophy are present. AP  is 210/140 mm  Hg.  What  is the most probable diagnosis?

Explanation

krushkrok No159 (2012)

krushkrok No159a (2012)

Cushing Syndrome: Etiology

*INCREASE Cortisol due to a variety of causes (Glucocorticoids).

*Exogenous corticosteroids:  result in DECREASE ACTH (MCC).

*Primary adrenal adenoma, hyperplasia or carcinoma (Cushing’s Syndrome).

*ACTH-secreting pituitary adenoma (Cushing Disease).

Findings: Hypertension, Weight Gain, Moon Facies, Truncal Obesity, Buffalo Hump, Skin Changes (thinning striae), Osteoporosis, Hyperglycemia (Insulin resistance), Amenorrhea, Immunosuppression.

160.

Microelectrode  technique   allowed  to register a potential following \\\"all-or-none\\\" law and being able  of  undecremental   spreading.  Specify this potential:

Explanation

Action potential is a series of electrical changes that occur in the membrane potential when the muscle or nerve is stimulated. It occurs in 2 phases: depolarization and repolarization. Excitable tissues obey the all-or-none law which states that when an excitable tissue is stimulated by a stimulus, it gives maximum response or does not give any response at all. Below the threshold level i.e. if the strength of stimulus is not adequate, the tissue does not give any response. Action potential obeys the all-or-none law because a particular threshold must be reached for an action potential to occur.

161.

A  patient has  been  diagnosed  with transmural  myocardial   infarction.   What drug should  be given in order  to prevent cardiogenic  shock?

Explanation

Some other options listed are opoid analgesics; but Promedol remains the best answer because it has a spasmolytic effect (antispasmodic). Therefore, it is very effective for pains associated with spasms of smooth muscles of internal organs and blood vessels. In comparison with morphine hydrochloride, it has less oppressing activity on respiratory centers, less than excites the center of the vagus nerve and the vomiting center. When the pain is associated with spasms of smooth muscle (angina pectoris, liver, kidney, intestinal cramps) you can assign promedol with atropine, metacin, papaverine etc.

162.

2   days   after    labour    a   woman developed  shock  along  with  DIC syndrome  that caused her death.  Autopsy revealed  purulent endomyometritis, regional  purulent lymphangitis,  lymphadenitis and  purulent thrombophlebitis. There were also dystrophic alterations and interstitial inflammation of  parenchymal organs. What is the most likely diagnosis?

Explanation

Septicemia is a form of sepsis. Hemorrhagic syndrome is well pronounced (petechial rash, hemorrhages to the serous and mucus membranes and internal organs, DIC). Hyperplasia of lymphoid and hemopoietic system is typical. Proliferation of lymphoid and reticular cells as well as accumulation of mature and immature blood cells are found in the spleen and lymph nodes. Interstitial inflammation develops  in the parenchymal organs (heart, liver, kidneys). Septicemia can lead to septic shock. DIC – disseminated intravascular coagulation

163.

A patient got an injury of spinal marrow   in  a  road  accident   that  caused loss of tactile sensation,  posture sense, vibration sense. What conduction tracts are damaged?

Explanation

Fasciculus gracilis (tract of Goll) and Fasciculus cuneatus (tract of Burdach): fasciculus gracilis and cuneatus are together called ascending posterior column tracts. These tracts are formed by the fibers from posterior root ganglia. Thus, both tracts are constituted by the fibers of first order neurons of sensory pathway. Functions: fine (epicritic) tactile sensation; tactile localization; tactile discrimination; sensation of vibration; conscious kinesthetic sensation; stereognosis (ability to recognize the known objects by touch with closed eyes).

 

Fasciculus gracilis contains the fibers from lower extremities and lower parts of the body. Fasciculus cuneatus contains fibers from upper part of the body.

164.

Examination of a patient revealed II grade obesity. It is known that he consumes  a lot  of sweets  and  rich  food, has sedentary way of life. That’s why anabolic  metabolism has  the  priority  in his organism. Which of the following pathways is amphibolic?

Explanation

The citric acid cycle (kreb’s cycle, TCA cycle) is a good example of amphibolic pathway. The first reaction of the cycle in which oxaloacetate (4 carbon compound) condenses with acetate (2 carbon compound) to form citrate (6 carbon compound) is typically anabolic. The next few reactions which are intramolecular rearrangements produce Isocitrate. The following two reactions are typically catabolic. ‘COO’ is lost in each step and succinate (4 carbon compound) is produced. Amphibolic involves both catabolism and anabolism.

165.

A  patient who  abuses  smoking  has chronic  bronchitis.  Biopsy of his primary bronchus  revealed  multilayer pavement epithelium.  What   pathological  process was revealed  in the bronchus?

Explanation

IMG_9796

Metaplasia is a reversible change of one type to another type of adult epithelial or mesenchymal cells, usually in response to abnormal stimuli and often reverts back to normal on removal of stimulus. Types (2):

·        Epithelial: *Squamous metaplasia: in bronchus (chronic smokers), cervix

  *Columnar metaplasia: there is transformation to columnar epithelium.

·        Mesenchymal metaplasia: osseous and cartilaginous.

Hyperplasia: ↑number of cells; Hypertrophy: ↑in size.

166.

A patient suffers from pulmonary tuberculosis. During  treatment neuritis  of visual nerve  arose. What  drug has caused this by-effect?

Explanation

Isoniazid is the hydrazide of isonicotinic acid and is a pyridine. Pyridine occurs in many important compounds including  azines and the Vitamins Niacin(B3) and Pyridoxine(B6). Therefore, isoniazid can be interfered with Vit.B3,B6 and even B1 metabolism by competing with them.

Isoniazid is a first line antituberculosis drug that inhibits the synthesis of mycolic acid. Vit B6 is needed for the transformation of tryptophan to Vit.B3.

Adverse reaction of Isoniazid: peripheral neuritis, optic neuritis, hepatitis and idiosyncratic hepatotoxicity.
167.

A  67 year  old  patient complains  of periodic  heart  ache, dyspnea  during  light physical activities. ECG reveals extraordinary contractions of heart ventricles. Such arrhythmia is called:

Explanation

Extrasystole is the premature contraction (an extra contraction) of the heart before its normal contraction. In other words, it is an extra contraction of the heart before its normal contraction. It is caused by an ectopic focus (discharge of an impulse from any part of the heart other than the Sinoatrial node).

 

Bradycardia: ↓heart rate; Tachycardia: ↑heart rate; Flutter: rapid heart contractions; Fibrillation: very rapid heart contractions. (In all this, there is no extra heart contraction, we just have faster contractions).

168. A patient with ischemic heart disease has   been   administered  an   anti-anginal drug that  reduces  the myocardial  oxygen consumption and  improves  blood  supply of myocardium. What drug is it?

Explanation

Nitroglycerine is an organic nitrate. By dephosphorylation of myosin chain; it reduces myocardial contractions → decreased myocardial O2 consumption; relaxation of vascular smooth muscle → ↑perfusion → improves blood supply to myocardium. FullSizeRender (34)

Administered nitrates → ↑ Nitrites →↑ Nitric oxide (NO) → ↑cGMP → ↑dephosphorylation of myosin chains → vascular smooth muscle relaxation. Nitroglycerin is an organic nitrates. Nitrates inhibit coronary vasoconstriction or spasm, increasing perfusion of the myocardium and thus relieving vasospastic angina. Because of this action, nitrates are effective in treating effort-induced (stress) angina. Classic, effort-induced or stable angina are experienced in such conditions as physical activity, emotional excitement or any other cause of increased cardiac workload.

169. A  doctor  asked  a patient to  breath out  fully  after  taking  a  normal   breath. What muscles contract during such exhalation?

Explanation

All muscles that elevate the rib cage are muscles of inspiration and those that depress the rib cage are muscles of expiration.

Muscles of inspiration:

·        Sternocleidomastoid: lift upward on the sternum.

·        Anterior serrati: lift many of the ribs.

·        Scalene: lift the first two ribs.

Muscles of expiration: Abdominal recti – pull down the lower ribs and other abdominal muscles also compress the abdominal contents upwards against the diaphragm and internal intercostals.

 

Normal respiration is accomplished by the movement of the diaphragm only.

170.

A  4 year  old  child  with  hereditary renal lesion has signs of rickets, vitamin D concentration in  blood  is normal.  What is the most probable cause of rickets development?

Explanation

Rickets results from insufficiency of vitamin D (calcitriol). Calcitriol is the active form of vitamin D. calcitriol production is dependent on the kidney’s 1-α-hydroxylase which converts 25-OHD3 to 1,25-(OH)2D3 (calcitriol). So in cases of renal lesion, there is lack of 1-α-hydroxylase which leads to a deficiency of calcitriol or impaired synthesis of calcitriol.

171.

A man was intoxicated with mushrooms. They contain  muscarine  that stimulates muscarinic cholinoreceptors. What symptoms  signalize intoxication with inedible mushrooms?

Explanation

Muscarinic receptors belong to the class of G protein-coupled receptors. These receptors, in addition to binding Acetylcholine, it also recognizes muscarine – an alkaloid that is present in certain poisonous mushrooms. Muscarinic effects include miosis, spasm of accommodation, increasing of lacrimal, salivary, gastrointestinal, bronchial and sweat glands secretion, increase of smooth muscle tonus of internal organs, bradycardia, decreased arterial pressure.

 

All other options are sympathetic effects (acting through α or β-adrenergic receptors).

172.

There  was a record  of some anthrax cases  among   animals   in  a  countryside. The  spread  of disease  can  be  prevented by means  of immunization. What  kind of vaccine should be used?

Explanation

Live spore vaccine (STI) is used for vaccination against anthrax. STI live vaccine
173.

An infectious  disease unit admitted a patient with signs of jaundice  caused  by hepatitis virus. Select an indicator that  is specific only for parenchymatous jaundice:

Explanation

In viral hepatitis, there is generalized liver dysfunction involving uptake and conjugation of unconjugated bilirubin, secretion of conjugated bilirubin into bile ducts, and recycling of urobilinogen. Alanine transaminase (ALT) and Aspartate transaminase (AST) are increased (↑), but ALT is higher than AST and there is a slight ↑ in alkaline phosphatase (ALP) and ɣ-glutamyltransferase (GGT).

ALT is a specific enzyme for liver necrosis; present in the cytosol. ALT>AST: viral hepatitis.

 

AST is present in the mitochondria. Alcohol damages mitochondria AST>ALT indicates alcoholic hepatitis

174. A  patient  presents with  dysfunction of cerebral cortex accompanied by epileptic  seizures.  He  has  been  administered  a biogenic amine  synthetized from glutamate and responsible for central inhibition.  What substance  is it?

Explanation

image
175.

A 50 year old patient underwent resection  of  tumour  of  large   intestine wall. Microscopically  it presents itself as fascicles of divergent collagen fibers of different thickness  and form and some monomorphous fusiform cells that are irregularly  distributed among  the  fibers. Cellular    atypia   is   not   evident.    What tumour is it?

Explanation

Fibroma is a benign connective (fibrous) tissue tumors. It can be hard (dense) or soft fibroma. Hard fibroma has fibrous connective tissue (e.g. collagen fibers) prevail over the cellular elements. Soft/loose fibroma has more cells than the connective tissue component.

In the question; fascicles of divergent collagen fibers with SOME fusiform cells irregularly distributed among the fibers i.e. fibers > cells (fibers dominate).

176.

Toxic affection  of liver results in dysfunction of protein synthesis. It is usually accompanied by the following kind of dysproteinemia:

Explanation

Liver is the primary organ of protein synthesis. In a case of dysfunction of protein synthesis, hypoproteinemia occurs. We have absolute and relative hypoproteinemia. Absolute is when there is disturbed synthesis (absolute hypoproteinemia) and in cases of increased synthesis (absolute hyperproteinemia); while relative does not have to do with synthesis but other pathologies e.g. renal lesions, in which case, there is loss of protein with urine (relative hypoproteinemia); also vomiting, diarrhea, profuse sweating can produce relative hyperproteinemia.

177.

A cell at the stage of mitosis anaphase was stimulated by colchicine  that  inhibits  chromosome separation  to  the  poles. What type of mutation will be caused?

Explanation

When separation does not occur, it is called non-disjunction and both members of a pair of homologous chromosome move into one cell. In mitotic non-disjunction, mosaicism is produced with some cells having an abnormal chromosome number and others being normal; an extra chromosome is present (trisomy – polysomy - polyploidy) or one is missing (monosomy). Polyploidy occurs when the entire chromosome set is present in more than two copies, so the individual may be triploid rather than diploid and have a chromosome number of 69. 
178.

Examination  of  duodenal contents revealed   some   pyriform   protozoa  with twin  nuclei   and   four   pairs   of  flagella. There  were two supporting filaments between the  nuclei  and  a  suctorial  disc on  the  ventral  side. What  representative of protozoa was revealed  in this patient?

Explanation

imageimage

Giardia lamblia is pear shape or “clownface” with two nuclei, 4 pairs of flagella. The two nuclei are outlined by adhesive discs. Transmitted by cysts in water. The cysts are oval, have 4 nuclei and have clearly visible axostyles. It causes Giardiasis – bloating, flatulence, foul-smelling, fatty diarrhea. Diagnosis: trophozoites or cysts in stool.

179.

In order  to prevent massive haemorrhage in the  region  of oral  cavity floor  it is required to  ligate  an  artery which is located  within Pirogov’s triangle. What artery is it?

Explanation

krushkrok No97 (2007)

The lingual artery arises at the level of posterior horn of the hyoid bone from the external carotid artery. The artery occupies the lingual triangle (of Pirogov) immediately below the hyoglossus muscle. It gives the dorsal lingual branches, deep lingual artery and sublingual artery.

180.

A patient with high-titer antinuclear antibodies died from progressing renal impairment. Autopsy revealed   mesangioproliferative glomerulonephritis and a bacterial polypous endocarditis. There was periarterial bulbar sclerosis in spleen and productive proliferative vasculitis  in skin. What is the most likely diagnosis?

Explanation

Systemic lupus erythematous (SLE, Libman-sacks disease) is the classic prototype of the multisystem disease of autoimmune origin, characterized by a bewildering array of autoantibodies, particularly antinuclear antibodies. It is characterized principally by injury to the skin, joints, kidney, and serosal membranes. Antinuclear antibody is directed against several nuclear antigens and can be grouped into 4 categories:

*       * Antibodies to DNA

·       * Antibodies to histones

·        *Antibodies to nonhistone proteins bound to RNA

·        *Antibodies to nuclear antigens.

 

SLE is a type III hypersensitivity reaction with formation of immune complexes. It can cause diffuse proliferative glomerulonephritis seen under the light microscope as “wire loop” of capillaries and granular under the immunofluorescence microscopy.

181.

A    shepherd   who    has    tended sheep   together  with   dogs   consulted   a doctor  about  pain  in  his right  subcostal area, nausea, vomiting. Roentgenoscopy revealed  a  tumour-like formation. What kind of helminthiasis might be suspected?

Explanation

·        Echinococcus granulosus causes Echinococcosis. It is composed of a scolex and only 3 proglottids, making it one of the smallest tapeworms. The scolex has a circle of hooks and 4 suckers similar to Taenia solium. Dogs are the most important definitive hosts. The intermediate hosts are usually sheep. Humans are almost always dead-end intermediate hosts. Transmission: ingestion of eggs from dog faeces. Disease – hydatid cysts in liver causing anaphylaxis if antigens released.

·        Diphyllobothrium latum causes diphyllobothriasis. A fish tapeworm. In contrast to the other cestodes, which have suckers , the scolex of D. latum has two elongated sucking grooves by which the worm attaches to the intestinal wall. Infection by D. latum causes little damage in the small intestine. In some individuals, megaloblastic anemia occurs as a result of vitamin B12 deficiency caused by preferential uptake of the vitamin by the worm. Transmission: ingestion of larvae from raw freshwater fish. Caviar is prepared from fish.

·        Taeniasis: there are two important human pathogens in the genus Taenia: T. solium (pork tapeworm) and T. saginata (beef tapeworm)

·        Trichiniasis (Trichinosis) is caused by Trichinella spiralis (nematode- roundworm). Transmission: fecal-oral; undercooked meat (especially pork). A few days after eating undercooked meat, usually pork, the patient experiences diarrhea followed by 1-2weeks later by fever, muscle pain, periorbital edema and eosinophilia.

 

·        Ascaridiasis (Ascariasis) caused by Ascaris lumbricoides (giant roundworm). The major damage occurs during larval immigration rather than from the presence of the adult worm in the intestines. The principal sites of tissue reaction are the lungs, where inflammation with an eosinophilic exudates occurs in response to larval antigens. Ascaris pneumonia with fever, cough and eosinophilia can occur with a heavy larval burden.

182.

For  relief  of  hypertensive  crisis  a doctor   administered  a   patient  a   drug that   apart   from  antihypertensive  effect has also sedative, spasmolytic and anti-convulsive effect. The drug was taken parenterally. When  it  is  taken   enterally it acts as a laxative and cholagogue.  What drug was administered?

Explanation

Allochol and cholenzyme are agents of biliary acids and bile (cholesecretics – induce bile secretion). Cholosas is a plant agent (cholesecretics – induce bile secretion). Nicodinum is a synthetic agent (cholesecretics – induce bile secretion). They all stimulate bile formation. Magnesium sulfate is a drug causing gall bladder contraction and relaxation of sphincter of oddi; it promotes bile excretion. It is also a salt laxative acting on all intestine’s secretions. It is a non-absorbable salt that hold water in the intestine by osmosis and distend the bowel, increasing intestinal activity and producing defecation.

183. A bacteriological laboratory received sputum  sample of a patient suffering from tuberculosis. Bacterioscopic examination of  smears  and  detection of  tuberculosis bacillus can be realized by one of enrichment  methods  that  involves processing of  sputum  only  with  solution  of  caustic soda. What is this method called?

Explanation

In laboratory diagnosis of tuberculosis, the nature of specimen depends on the site of lesion. In pulmonary tuberculosis, 3 specimens are investigated – sputum, laryngeal swab and gastric savage. Direct microscopy of a suitably stained Ziehl Neelson smear may reveal the presence of tubercle bacilli. The sputum for microscopy should always be subjected to concentration method which serves 3 purposes – concentration, homogenization and decontamination. Petroff’s method using caustic soda (4%) yields good results.  Homogenization may also be done by sulphuric acid, oxalic acid or trisodium phosphate.

184.

Electronic microphotography of pulmonary alveoli’s wall presents a big cell. Its cytoplasm has a lot of mitochondria, developed Golgi apparatus, osmiophil lamellated corpuscles.  What is the main function of this cell?

Explanation

Alveolar epithelium is composed of type I and II alveolar cells and occasional brush cells. Type II alveolar cells or type II pneumocytes or septal cells are secretory cells. Their apical cytoplasm is filled with granules as stacks of parallel membrane lamellae, the lamellar bodies. They are rich in a mixture of phospholipids, neutral lipids and proteins that is secreted by exocytosis to form an alveolar lining, surface-active agent called surfactant.

185. A patient with diabetes mellitus complicated by angiopathy has been recommended a drug which is a sulphonyl urease   derivate of  the  second  generation. It improves microcirculation and is known for its relatively good tolerance. What drug is it?

Explanation

Sulfonylurea are insulin secretagogues because they promote insulin release from the β-cells of the pancreas. The primary drugs used today are the 2nd generation drugs – glibenclamide, glipizide, glimepiride etc. These agents are useful in the treatment of patients who have type 2 diabetes mellitus and cannot be managed by diet alone. Metformin is the only currently available biguanide; its classed as an insulin sensitizer. Acarbose – α glucosidase inhibitor. Butamide (tolbutamide) is also a sulfonylurea but is older and glibenclamide is more potent and used much more commonly. Actrapid (soluble insulin) is fast acting but not a sulfonylurea.

186.

A child is pale, pastose, muscular  tissue  is bad  developed, lymph  nodes  are enlarged.  He   often   suffers   from   angina and pharyngitis, blood has signs of lymphocytosis.  The child is also predisposed to autoallergic diseases. What type of diathesis can be presumed in this case?

Explanation

Abnormality of constitution, so called diathesis is characterized by pathological reactions on physiological agent. Classification of diathesis:

·        Hemorrhagic: characterized by hemorrhagic reaction to physiological factors.

·        Thymicolymphatic/Lymphohypoplastic: characterized by enlargement of lymphatic nodes, muscular atrophy, individual pale, pastous, predisposing to autoallergic disease, lymphocytosis, angina and other infectious diseases.

·        Neuroarthritic/Gouty: predisposing to arthralgia, arthritis, rheumatism, obesity, gout, psychic disease.

·        Edematic/Exudative: characterized by edema reaction to different environmental factors.

 

·        Asthenic: expressed in hypodynamia, hypotonia.

187.

A  26  year  old  pregnant woman  is under   treatment at  an  in-patient hospital. After  a continuous attack  of vomiting she was found to have reduced volume of circulating  blood. What kind of change in general blood volume is the case?

Explanation

Polycythemic hypovolemia is a condition with a decreased amount of plasma usually characterized by appreciable concentration and increased viscosity of blood. Hematocrit is above normal. This condition is observed in connection with considerable loss of water by the organism as in diarrhea, intractable vomiting, extensive burns. Hematocrit is increased because there is no loss of erythrocytes just fluid was lost from vomiting. It can’t  be hypervolemia because there is loss of fluid from prolonged vomiting.

 

In simple hypovolemia: hematocrit is normal; oligocythemic hypovolemia: hematocrit is decreased.

188. Vomiting matters of a patient suspected of having cholera were delivered to the bacteriological laboratory. The  material was  used  for  preparing a \\\"hanging  drop\\\"specimen. What type of microscopy will be applied for identification of the causative agent by its mobility?

Explanation

Vibrio cholera is the cause of cholera. It is curved, comma-shaped, gram negative rods. It is transmitted by fecal contamination of water and food, primarily from human sources. Clinically, watery diarrhea in large volumes is the hallmark of cholera. There are no red blood cells or white blood cells in the stool. Rice-water stool is the term often applied to the non-bloody effluent. Grows in an alkaline media. Dark-field and phase contrast microscopy have been used for screening fecal specimens for the presence of V. cholera. With these techniques, liquid stools are microscopically examined for the presence of organisms with typical darting (“shooting star”) mobility [analogous to ‘hanging drop’].

189. Examination of a 6-month-old child revealed  a delay in closure  of the  occipital  fontanelle. When  should  it  normally close?

Explanation

font....

Fontanelles are large fibrous areas where several sutures meet; often called “soft spots” on an infant’s head. The two largest fontanelles are the anterior and posterior fontanelles on the superior surface of the neurocranium. The fontanelle in the back of the head (posterior fontanelle) usually closes by the time an infant is 2-3 months old. The fontanelle at the top of the head (anterior fontanelle) usually closes between 7-18months. In adults the remnants of the anterior fontanelle is the bregma and the posterior fontanelle is the lambda.

190. A  45 year  old  male  died  from  disseminated tuberculosis. On  autopsy  the symptoms  of tuberculosis were confirmed by both microscopical and histological analyses. All the affected organs had epithelioid cell granulomas with caseous necrosis in the centre. What kind of hypersensitivity reaction underlies the process of granuloma development?

Explanation

FullSizeRender (7)

Mantoux test is a type IV Hypersensitivity reaction (HSR), which involves macrophages,T-lymphocytes and lymphokines(cytokines). Mononuclear cells (lymphocytes,monocytes,macrophages).

Remember,it is antibody independent (i.e does not involve antibodies).

B-lymphocytesàPlasma cellsàIg(Antibodies)------- none is involved in Type IV HSR.

Type IV (cell mediated, delayed): antibody-independent T-cell mediated reactions e.g. positive mantoux reaction (tuberculin test), hashimoto’s thyroiditis or transplant rejection etc.

The Mantoux skin test should be read between 48 and 72hrs after administration. The basis of reading  is the presence or absence of induration, which may be determined by inspection and by palpation. A record should also be made of formation of vesicles, bullae, lymphangitis, ulceration and necrosis at the test site. The formation of vesicles, bullae or necrosis at the test site indicates positive result. A negative mantoux result usually signifies that the individual has never been exposed to Mycobacterium tuberculosis i.e. absence of cell mediated immunity to tuberculin.

191.

A male patient has been  diagnosed with  acute  radiation disease.  Laboratory examination revealed a considerable reduction of platelet serotonin level. The likely cause of platelet serotonin reduction is the disturbed metabolism of the following substance:

Explanation

Serotonin, also called 5-hydroxytryptamine, is synthesized and stored at several sites in the body. It can be found in the intestinal mucosa, central nervous system and in platelets. Serotonin is synthesized from tryptophan, which is hydroxylated. The product, 5-hydroxytryptophan is decarboxylated to serotonin, which is also degraded by monoamine oxidase (MAO). Serotonin has multiple physiologic roles, including pain perception, affective disorders, and regulation of sleep, temperature and blood pressure.

192.

In course of a conditional experiment the   development of  mesenchyma  cells was  completely   inhibited.   Development of the following muscular tissue will be disturbed:

Explanation

Mesenchyme refers to loosely organized embryonic connective tissue regardless of origin. Undifferentiated embryonic mesenchymal cells are round/cuboidal in shape. During development, visceral myogenesis is shortly preceded by mesenchymal cell elongation. Undifferentiated embryonic mesenchymal cells from intestine (abundant visceral muscle), lung (some visceral muscle) or kidney (no visceral muscle); these cells differentiate into smooth muscle upon elongation.

193. Sex chromosomes of a woman didn’t separate and move  to the  opposite  poles of a cell during  gametogenesis (meiosis). The ovum was impregnated with a normal spermatozoon. Which chromosomal disease can be found in her child?

Explanation

image

krushkrok No105 (2014)

In meiosis, two members of a pair of homologous chromosomes normally separate during the first meiotic division, so that each daughter cell receives one member of each pair. Sometimes, however, separation does not occur (nondisjunction) and both members of a pair move into one cell. As a result of nondisjunction of the chromosomes, one cell receives 24 chromosomes and the other receives 22 instead of the normal 23. When at fertilization, a gamete having 23 chromosomes (spermatozoon) fuses with a gamete having 24 or 22 chromosomes, the result is an individual with either 47 chromosomes (47 XXX – trisomy X) or 45 chromosomes (45 XO – monosomy X, Turner’s syndrome). In women, the incidence of chromosomal abnormalities including nondisjunction, increases with age especially at 35years and older.

194. After taking poor-quality food a patient  developed repeated  episodes   of  diarrhea. On the next day he presented with decreased arterial pressure, tachycardia, extrasystole. Blood pH is 7,18. These abnormalities were caused by the development of:

Explanation

Acidosis is the reduction in pH (increase in H+ concentration) below normal range. pH is less than 7.35; it is produced by:

·        Increase in partial pressure of CO2 in the body.

·        Decrease in HCO3- concentration.

Alkalosis is the increase in pH (decrease in H+ concentration) above normal range. pH is greater than 7.45; it is produced by:

·        Decrease in partial pressure of CO2 in the body.

·        Increase in HCO3- concentration.

Each of these two disorders has respiratory and non-respiratory forms. The non-respiratory form is divided into metabolic and excretory(non-gaseous).

·        Respiratory acidosis is the acidosis that is caused by alveolar hypoventilation e.g. airway obstruction due to bronchitis or lung diseases (pneumonia).

·        Respiratory alkalosis is caused by alveolar hyperventilation e.g. hypoxia in high altitude.

·        Non-respiratory:

-Metabolic acidosis is characterized by excess accumulation of organic acids such as lactic acid, ketoacids and uric acid formed by normal metabolism e.g. in Diabetes mellitus or extreme/prolonged exercise.

-Excretory/Non-gaseous acidosis may develop in impaired renal H+ excretion related to increased loss of bicarbonate in urine; diarrhea causes acidosis by the loss of bicarbonate with faeces.

-Excretory/Non-gaseous alkalosis: vomiting (loss of gastric acid), increased metabolism of lactate and citrate (turns into bicarbonate and water), long-term use of thiazides and loop diuretics.

 

      It is excretory/non-gaseous alkalosis because of the frequent vomiting.

195.

A patient has been diagnosed with influenza.  His condition became   drastically worse after taking antipyretic drugs. His consciousness is confused, AP is 80/50 mmHg, Ps  is 140/m, body temperature dropped down to 35, 8oC. What complication developed in this patient?

Explanation

His condition became worse after taking antipyretic drug. Antipyretic drugs block prostaglandin synthesis by inhibiting cyclooxygenase (COX) enzyme at the thermoregulating centers in the hypothalamus and at peripheral target sites. In case of toxicity of this drug, it can result in coma followed by cardiovascular collapse and respiratory arrest due to its CNS depressant activity. Consciousness is confused; arterial blood pressure is low and pulse rate is very high – these are clear signs of collapse.

196.

One  of  sections  of  central  nervous system has layerwise arrangement of neurocytes. Among  them  there  are  cells of the  following forms: stellate,  fusiform, horizontal, pyramidal. What section of central  nervous  system is this structure typical for?

Explanation

image

Cerebral cortex consists of gray mater that surrounds the deeper white mater. The cerebral cortex is formed by 6 layers of structures:

·        Molecular or plexiform layer or horizontal layer ( contains horizontal cells of cajal).

·        External granular/stellate layer

·        Outer pyramidal layer

·        Internal granular/stellate layer

·        Internal pyramidal layer

·        Fusiform cell layer

Layers of grey mater in cerebellum:

·        Outer molecular or plexiform layer

·        Intermediate purkinje layer

·        Inner granular layer

197.

Pharmacological effects of anti- depressants are based upon blocking (inhibiting) the   enzyme   that   acts  as  a catalyst for the breakdown of biogenic amines noradrenalin and serotonin in the mitochondria of cephalic  neurons. What enzyme takes part in this process?

Explanation

image

Norepinephrine (noradrenaline), epinephrine (adrenaline) and dopamine are catecholamines produced in chromaffin cells of adrenal medulla from tyrosine. The catecholamines are inactivated by oxidative deamination catalyzed by monoamine oxidase (MAO) and by O-methylation carried out by Catechol-O-methyltransferase. The metabolic products of these reactions are excreted in the urine as vanillymandelic acid from epinephrine and norepinephrine; and homovanillic acid from dopamine.

198. During    examination   of   a   patient  a  dentist   revealed   a  lot  of  \"white spots  zones  of enamel  demineralization. What microorganisms take part in the development of this process?

Explanation

Streptococcus mutans are gram positive cocci shaped bacteria. They are facultative anaerobes and a major contributor of tooth decay. It’s a cariogenic microorganism that breaks down sugar for energy and produces an acidic environment which demineralizes the superficial structure of the tooth. The result of the conversion disintegrates the coating of the tooth then later dissolves the calcium molecule creating a hole. Transmission of S. mutans can be found in people of all ages although it is more common for infants and children. Since every human has bacteria in their mouth, the only prevention is to lessen the impact of acid fermentation by practicing adequate oral hygiene.

199. A 4  year  old  child  presents  with general weakness, sore throat and deglutitive  problem.  After   his examination a doctor suspected  diphtheria and sent  the material to the bacteriological laboratory.  In order to determine the diphtheria causative agent the material should be inoculated into the following differential diagnostic medium:

Explanation

Diphtheria bacteria (Corynebacterium diphtheria) is Gram positive, pleomorphic, often club-shaped rods and are arranged in palisades or in V (at an angle) or L-shaped formations. Media used for isolation are Tellurite agar & Lὄffler medium. Lὄffler nutrient medium consists of coagulated serum & nutrient broth. Selective indicator medium containing tellurite are used in selective culturing. K tellurite is used to inhibit the accompanying flora.
200.

A man with a stab wound in the region of the quadrilateral foramen consulted a doctor about it. Examination revealed that the injured  couldn’t abduct  his arm from the  body. What nerve is most likely damaged?

Explanation

krushkrok No28 (2013)

The axillary nerve (nervus axillaris) from brachial plexus, is the greatest branch of the short branches of brachial plexus. It arises from the posterior cord and proceeds to the quadrangular/quadrilateral foramen. It supplies the deltoid and teres minor muscles. The axillary nerve and posterior circumflex humeral artery pass through the quadrangular space. Borders of the quadrangular space:

·        Superior: Teres minor

·        Inferior: Teres major

·        Medial: Long head of triceps

·        Lateral: humerus